Collections: Total Generalship: Commanding Pre-Modern Armies, Part II: Commands

This is the second of a three-part (I, II, IIIa, IIIb, IIIc) look at the role of the general in a pre-modern army, particularly in the context of a pitched battle. Last week, we looked at the information a general might have before and during a battle. What we found was that, in contrast to the broadly omniscient generals of film and video games, actual generals had to work in an information environment which was both unsure (that is, information was frequently false) and substantially incomplete.

This week, we’re going to turn to look at what the general can do with that information, focusing on his actions during the battle itself. As we’ll see, the role of the general during the battle can be divided effectively in two parts, which we might term ‘command’ and ‘leadership.’ ‘Command’ is the thing that most video games simulate: the specific tactical choices the general makes concerning the disposition of his army, its movement and so on. But as important – often more important – was the general’s leadership role: inspiring his troops, performing the role of general to their satisfaction and giving them a sense of confidence in the possibility of victory. While popular culture often focuses more on command, in pre-modern armies, leadership was, as we’ll see, often the dominant factor (though command is by no means absent!).

At the same time it is focused on command, the popular vision of it dispenses with many of its inherent limitations. I suspect every military historian who has ever discussed a pre-modern battle with a lay audience has gotten some version of “well why didn’t they do X” where ‘X’ was a battle-plan far too intricate, too finicky and complex and most importantly required too much agility in command – that is, too much ability to change the plan ‘on the fly’ – to ever be implemented. Often they are plans with so many moving parts that one wonders if they could be accomplished with modern communications equipment. But such plans are not only possible in strategy video games, they are often optimal (and are a staple of the ‘chessmaster‘ fiction trope). So let’s talk about why even the very best generals had only very limited control of an army in motion.

And as always, if you like what you are reading here, please share it; if you really like it, you can support me on Patreon. And if you want updates whenever a new post appears, you can click below for email updates or follow me on twitter (@BretDevereaux) for updates as to new posts as well as my occasional ancient history, foreign policy or military history musings.

Point and Click Command

Of course in a game like Total War this is all very easy; you click on your unit and then you click where you want it to go. The unit responds almost instantly and understands your orders exactly. Indeed, when the latest Total War game, the fantasy-themed Total War: Warhammer III, released there was a bug where some units (mostly missile infantry) would take one or two extra seconds to respond to orders. This was a major problem! Creative Assembly had to rush to fix it.1

As we’ll see, in a real battle when seconds count, new orders are only a few minutes away. Well, sometimes they’re rather more than a few minutes away. Or not coming at all.

This is also true, of course, in films. Our friend Darius III here below from Alexander (2004) silently waves his hand to mean ‘archers shoot!’ and also ‘chariots, charge!’ and then also ‘everyone else, charge!’ Keeping in mind what we saw about the observation abilities of a general on horseback, you can well imagine how able Darius’ soldiers will have been to see his hand gestures while they were on foot from a mile or so away. Yet his army responds flawlessly to his silent arm-gestures. Likewise the flag-signalling in Braveheart‘s (1995) rendition of the Battle of Falkirk: a small banner, raised in the rear is used to signal to soldiers who are looking forward at the enemy, combined with a fellow shouting ‘advance.’2 One is left to assume that these generals control their armies in truth through telepathy.

Touchdown! Wait, no, actually, I mean charge! Well, some of you. Not you. Or him. But that guy – Hey You! Get running! No, not you, the OTHER you.

There is also never any confusion about these orders. No one misinterprets the flag or hears the wrong orders. Your unit commanders in Total War never ignore or disobey you; sure the units themselves can rout, but you never have a unit in good order simply ignore your orders – a thing which happened fairly regularly in actual battles!3 Instead, units are unfailingly obedient right up until the moment they break entirely. You can order untrained, unarmored and barely armed pitchfork peasant levies to charge into contact with well-order plate-clad knights and they will do it.4

The result is that battleplans in modern strategy games are often impressive intricate, involving the player giving lots of small, detailed orders (sometimes called ‘micro,’ short for ‘micromanagement’) to individual units. It is not uncommon in a Total War battle for a player to manually coordinate ‘cycle-charges’ (having a cavalry unit charge and retreat and then charge the same unit again to abuse the charge-bonus mechanics) while also ordering their archers to focus fire on individual enemy units while simultaneously moving up their own infantry reserves in multiple distinct maneuvering units to pin dangerous enemy units while also coordinating the targeting of their field artillery. Such attacks in the hands of a skilled player can be flawlessly coordinated because in practice the player isn’t coordinating with anyone but themselves.

And I think most people have an intuitive sense that battlefield command didn’t work like this but it is worth discussing why because of how that ‘why’ places limits on a general’s ability to micromanage his battles.

Performing Generalship

We have to start not with tactics or the physics of shouting orders, but with cultural expectations. First, we need to establish some foundations here. First, in a pre-modern battle (arguably in any battle) morale is the most critical element of the battle; battles are not won by killing all of the enemies, but by making the enemies run away. They are thus won and lost in the minds of the soldiers (whose minds are, of course, heavily influenced by the likelihood that they will be killed or the battle lost, which is why all of the tactics still matter). Second, and we’ve actually discussed this before, it is important to remember that the average soldier in the army likely has no idea if the plan of battle is good or not or even if the battle is going well or not; he cannot see those things because his vision is likely blocked by all of his fellow soldiers all around him and because (as discussed last time) the battlefield is so large that even with unobstructed vision it would be hard to get a sense of it.

So instead of assessing a battle plan – which they cannot observe – soldiers tend to assess battle commanders. And they are going to assess commanders not against abstract first principles (nor can they just check their character sheet to see how many ‘stars’ they have next to ‘command’), but against their idea of what a ‘good general’ looks like. And that idea is – as we’re about to demonstrate – going to be pretty dependent on their culture because different cultures import very different assumptions about war. As I noted back in the Helm’s Deep series, “an American general who slaughtered a goat in front of his army before battle would not reassure his men; a Greek general who failed to do so might well panic them.” An extreme example to be sure, but not an absurd one. In essence then, a general who does the things his culture expects from him is effectively performing leadership as we’ve defined it above.

But the inverse of this expectation held by the soldiers is that generals are not generally free to command however they’d like, even if they wanted to (though of course most generals are going to have the same culturally embedded sense of what good generalship as as their soldiers). Precisely because a general knows his soldiers are watching him for signs that he is their idea of a ‘good general,’ the general is under pressure to perform generalship, whatever that may look like in this cultural context. That is going to be particularly true because almost all of the common models of generalship demand that the general be conspicuous, be available to be seen and observed by his soldiers. As a result, cultural ideals are going to heavily constrain what the general can do on the battlefield, especially if they demand that the general engage personally in combat.

Different Sorts of Generals

We can actually get a sense of a good part of the range simply by detailing the different expectations for generalship in ancient Greek, Macedonian and Roman societies and how they evolved (which has the added benefit of sticking within my area of expertise!).5

On one end, we have what we might call the ‘warrior-hero general.’ This is, for instance, the style of leadership that shows up in Homer (particularly in the Iliad),6 but this model is common more broadly. For Homer, the leaders were among the promachoi – ‘fore-fighters,’ who fought in the front ranks or even beyond them, skirmishing with the enemy in the space between their formations (which makes more sense, spatially, if you imagine Homeric armies mostly engaging in longer range missile exchanges in pitched battle like many ‘first system‘ armies).

The idea here is not (as with the heroes of Homer) that the warrior-hero general simply defeats the army on his own,7 but rather that he is motivating his soldiers by his own conspicuous bravery, ‘leading by example.’ This kind of leadership, of course, isn’t limited to just Homer; you may recall Bertran de Born praising it as well:

And I am as well pleased by a lord
when he is first in the attack,
armed, upon his horse, unafraid,
so he makes his men take heart
by his own brave lordliness.

On the opposite end of the spectrum, there is the pure ‘general as commander’ ideal, where the commanding general (who may have subordinates, of course, who may even in later armies have ‘general’ in the name of their rank) is expected to stay well clear of the actual fighting and instead be a coordinating figure. This style, for reasons that we’ll get to a little later in this post, is fairly rare in the pre-gunpowder era, but becomes common afterwards. Because in this model the general’s role is seen primarily in terms of coordinating various independently maneuvering elements of an army; a general that is ‘stuck in’ personally cannot do this effectively. And it may seem strange, but violating these norms with excessive bravery can provoke a negative response in the army; confederate general Robert E. Lee attempted to advance with an attack by the Texas Brigade at the Battle of the Wilderness (May 6, 1864) only to have his own soldiers refuse to advance until he retired to a more protected position.8 Of course this sort of pure coordination model is common in tactical video games which only infrequently put the player-as-general on the battlefield (or even if the ‘general’ of the army is represented on the battlefield, the survival of that figure is in no way connected to the player’s ability to coordinate the army).

In practice, pre-modern (which is to say, pre-gunpowder) generals almost never adopt this pure coordination model of generalship. The issue here is that effective control of a gunpowder army both demands and allows for a lot more coordination. Because units are not in melee contact, engagements are less decisive (units advance, receive fire, break, fall back and then often reform to advance again; by contrast a formation defeated in a shock9 engagement tends not to reform because it is chased by the troops that defeated it), giving more space for units to maneuver in substantially longer battles. Moreover, units under fire can maneuver, whereas units in shock generally cannot, which is to say that a formation receiving musket or artillery fire can still be controlled and moved about the field, but a unit receiving sword strikes is largely beyond effective command except for ‘retreat!’

In between these two extremes sits variations on what Wheeler (see footnote above) terms a ‘battle manager,’ which is a bit more complex and we’ll return to it in a moment.

What I want to note here is that these expectations are going to impact where the general is on the battlefield and thus what he can do to exert command. A general in a culture which expects its leaders to be at the front leading the army has the advantage of being seen by at least some of his soldiers (indeed that is the point – they need to see him performing heroic leadership), but once engaged, he cannot go anywhere or command anyone. This is also true, by the by, in cultures where the general is expected to be on foot to show that they share in the difficulties and dangers of the infantry; this is fairly rare but for much of the Archaic and Classical periods, this was expected of Greek generals. Even if a general on foot isn’t in combat directly, their ability to see or move about the battlefield is going to be extremely limited.

On the flipside, a general who is following the ‘commander’ ideal is likely to be in the rear, perhaps in an elevated position for observation. The obvious limitation here is that such a commander is going to struggle to display leadership because no one can see them (everyone is facing towards the enemy, after all). But that also impacts their ability to command – no one is looking at them so if they want to change their plans on the fly they need to send word somehow to subordinate officers who are with or in front of the battle line who can then use their visibility to communicate those orders to the troops.

And those communication problems bring us to:

Signals and Shouting

Let us assume, picking up our discussion of information last time, that our army is formed up into its battle array (pre-planned the night before, recall) and is advancing and our general has just now noticed something that demands a change in the plan. It could be a dangerous enemy attack (perhaps on the flank) or an opportunity to split the enemy line. Whatever it is, our general needs to make some alteration to the battle plan. It is almost certainly a fairly minor alteration, as with a battle line anywhere from a kilometer to several kilometers long, it would, for instance, take far too long to shuffle the right-to-left order of the line just due to the marching time involved. Nevertheless, the general needs to issue an unplanned, on-the-spot command; how does he do it?

The first option, of course, is shouting. The problem here is obvious: how is the commander’s command to be heard? Interestingly, there has been a fair bit of research by ancient historians looking at the question of how many people can possibly hear a short address unaided by modern loudspeakers and the like; figures vary but generally a few thousand if they are reasonably compact and quiet.10 That might work for a general’s pre-battle speech, delivered before the army advances, but it will not do for an army that is already in motion, much less once the chaos of battle has begun. Thousands of men marching (let alone fighting!) are noisy!

The modern solution to this problem is radio, but of course that’s hardly available to our pre-modern commander. Instead, to judge by films, the mind quickly jumps to signal flags. I am reminded of Braveheart (1995)’s rendition of the Battle of Falkirk, where Edward I uses signal flags to order his archers forward. HBO’s Rome also does this in its version of the Battle of Philippi, with flags being jostled and then pointed forward to signal the advance. Unfortunately, signal flags – as distinct from unit flags (which we’ll come back to in a moment) – have a few key problems, the most notable of which is that no one will be looking at them: after all the army is advancing, the soldiers are looking forward but signal flags (again, as opposed to unit flags) are going to be behind them, not placed out in the middle of No Man’s Land between the armies. As a result, signal flags are useful for sending information long distances (in a chain of stations or operators), for instance from one commander at distance to another, but not in battle; operational, rather than tactical tools. In practice, the use of signal flags like this is confined to the modern era; the first successful ‘optical telegraphs’ (as iterations on things like smoke signals and fire relays) date to the late 18th century.

Via Wikipedia, a relief from the Column of Trajan (c. 106) showing the legionary signa, in this case three aquila, on the left. Though the artist has not drawn this out, each legion’s aquila would have been somewhat distinct within the general pattern here.

Unit flags – a banner or other big, obvious symbol (like a statue of an eagle on a stick) – are more useful. These can be positioned at the front of a unit, typically at its center. If it advances, then the soldiers in the unit also know to advance, following the standard they can see (because it is elevated, large and visible) even if they cannot hear the orders. There are two complications here though: first, the unit banner or flag is a relatively late innovation in antiquity, really only coming into its own with the Romans. The Achaemenids may have used some kinds of ensigns or standards, but the Greeks do not seem to have done. Instead our first really good documentation of something like a battle flag comes from the Romans: each legion had a signa (eventually standardized to the legionary eagle, the aquila), which was a shiny metal statue mounted on a poll so it could be easily seen. Units of the legion broken off to do other things might instead follow a less impressive cloth banner, a vexillum, by which such detachments became known as vexillationes. But the broader problem is that of course your general may not be particularly close to your flags (or other standards) which are generally at the front-center of each component unit of your army. The flags may allow a subordinate officer to ‘drive’ the unit over the battlefield – and that’s good – but it doesn’t let the general tell that officer what to do.

Via Wikimedia Commons, relief probably from a triumphal Arch of Marcus Aurelius, now in the Capitoline Museum in Rome, showing the emperor on horseback in front of a pair of vexilla or detachment battle-flags.

A better option is music, but once again development seems to come fairly late in antiquity. Greek hoplites seem to have advanced to the music of the aulos, a double-reeded flute-like instrument; given the limitations of the instrument it is generally assumed it was used to keep time (so everyone marched in step) not transmit orders. Once again, a more complex system of musical signalling seems to come with the Romans, at least as detailed by Vegetius.11 Vegetius (2.22) notes three different kinds of horn instruments used by a legion: the tubicen was used to sound charge and retreat, the cornicen regulated the movement of the signa (so ‘advance’ or ‘halt’), while the buccina was used mostly for camp signals: sounding watches or assemblies. It’s a system that is akin to later bugle calls, but note that the orders it can give are limited to a relative handful of prearranged signals: advance, halt, charge, retreat, assemble, change shift and so on.

Via Wikipedia, a detail of the Chigi Vase (c.650 BC) showing an auletes (aulos-player) apparently keeping marching time for an advancing line of men in hoplite equipment.

The attentive reader here may have already noticed how developed Roman command and control is and may suspect that ties in with the Romans having a more ‘command’ oriented culture of generalship; if so you are ahead of the game!

Via Wikipedia, Roman military musicians (with cornicines) from the Column of Trajan (c. 106).

Of course if those instruments are sounding on a per-unit basis (and they are) that means you still have the problem of getting the order from the general to the instruments for the unit in question. And fundamentally here, the technology is – as I tell my students – man-on-horse. The particular fellow on the horse may be a dedicated messenger (if your military organization has those) or a subordinate officer or it may be the general himself.

But it is important to note now the limitations of this sort of system and we can use what we know of the Roman command and control system (as noted, one of the more developed of such systems prior to gunpowder) to get a sense of them. Let’s say the general realizes there is a problem on his flank and he needs a unit (probably here we’re talking a cohort or a maniple, not a legion) to change what it is doing. First off, the order needs to get within shouting range of the unit’s commander (in this case a senior centurion). The general can either go themselves or send a messenger; both options have their downsides. If the general goes himself he is essentially removing himself from observing or commanding the rest of the battle, but a common problem with sending a junior subordinate is that the unit commander may not respect or feel the need to obey that subordinate (written orders can help with this, but now we’re bringing in questions of literacy). Of course both a messenger or a general in transit may also well be killed, which will prevent the order from being received!12

In either case, the message is going to move at galloping speed, which is around 40km/h, meaning that it may take several minutes for the general or messenger to navigate to the spot. That doesn’t sound so bad, but battles with contact weapons do not typically go for hours and hours; Pydna (168) was, as noted last week, decided in about an hour total! Of course a battle might be longer (or shorter!) than this, though much of that extra time is likely pre-battle skirmishing – the actual direct press of infantry formations in shock rarely lasts long because of the terror of it (and to a lesser extent its lethality; we’ll return to the balance of terror and lethality next time). Imagine if you were playing a Total War game and your input delay was, say, five minutes long in a battle that might only last an hour or two.

But of course galloping time isn’t the end of it. The message now has to be conveyed to the unit. In the Roman system, that means the messenger needs to find the appropriate centurion, explain the order to him and then ideally that fellow will then signal the instruments and signa to act accordingly – but even then, those instruments and signa only have a handful of prearranged signals available. Anything more complicated will need to be shouted down the line the old fashioned way (as we know, for instance, the Spartans did for lack of almost any of the rest of this apparatus of command, Xen. Lac. 13.9). Needless to say that means that giving any complex order to a unit already engaged or about to be engaged is going to mean starting by signalling retreat and then attempting to regroup the unit; regrouping an already retreating unit is one of the most difficult tasks on a battlefield and is rarely performed successfully in an unplanned fashion (even in an planned fashion it goes wrong as often as it goes right).

(This is, by the by, why reserves are so important. An unengaged unit hanging behind the lines can be given new orders far, far easier than a unit that is already engaged or about to be. And indeed, those familiar with the Roman system of fighting with its three lines of heavy infantry will note that it is a system heavy on reserves. Indeed, the manipular legion essentially assumes it will be necessary to retreat and regroup the first line of heavy infantry (the hastati) behind the second (the principes) and plans and drills for that. Note how the Roman command culture, the Roman fighting method and the actual apparatus of messengers, signa, instruments and junior officers all align here – that’s common because these sort of institutions tend to co-evolve)

By contrast we may compare a Greek hoplite army in the Classical Period. It has no battle flags or ensigns and the general is expected to fight on foot. In the past I’ve described the resulting phalanx as an ‘unguided missile‘ and this is a big reason why. That’s not to say hoplite generals never exerted command on the battlefield – better generals might keep a reserve to be rushed to important points (as Pagondas does at the Battle of Delium in 424 BC). But for the most part, once a hoplite general formed up the army and hit ‘go,’ they had very little control over the army.

Being Seen

The modern instinct here is to look at the contrast between a ‘command’ oriented general and a ‘leadership’ oriented one and to wonder why any culture preferred from-the-front leadership instead of from-the-back command. This is in part because we are simply unconsciously preferring our own model of generalship, but it is also because modern warfare with its tremendous dispersion (things are much more spread out to avoid the massive firepower of modern weapons) makes visual leadership – being seen performing leadership – impossible beyond the smallest units.13

But remember: battles are not won by killing all of the enemies, but by making the enemy run away. Which means battles are principally won in the minds of the combatants. As a result, inspiring leadership matters. Now many games model this very crudely by just giving soldiers a morale boost based on their proximity to the general. But generals do not radiate a warm aura of comfort by their nature. In a real battle they need to be seen, humans being primarily visual creatures. A general conspicuous at the front of the army signals confidence and aggressiveness, which encouraged both in his soldiers.

Of course for that to work, the general must be recognized and we know that in almost all pre-modern armies there was a real concern for officers and the general in particular to be readily recognizable to their troops, even in situations where they were unlikely to be exerting a command function. In Greek and Roman armies, elaborate helmet crests (especially transverse – that is, left-to-right, crests) were often used to signal officers. Strategoi (generals) in Athens interesting seem to have stuck with the heavier Corinthian helmet (at least, to go by their statuary) long after most hoplites had gone to newer, lighter types. Distinguishing generals with archaic features also goes for the Romans: Roman generals are depicted in bronze or iron ‘muscle cuirasses’ and with ‘Attic’ type helmets, both pieces of equipment which by the second century BCE were long out of use in the rest of the army, which had adopted Gallic mail armor and a version of the Gallic ‘Montefortino’ type helmet. Nevertheless, muscle cuirasses and Attic helmets on Roman generals and later emperors persist well into the third century AD. Of course in the Middle Ages we see the development of elaborate systems of heraldry and banners to distinguish kings and nobles on the battlefield as well and it’s clear that kings and high nobility often also wore much more elaborately decorated armor so they would stick out and be easily seen.

Via Wikipedia, the famous bust of Pericles (495-429 BC), a Roman copy of a Greek original c. 430 BC. The Corinthian helmet Pericles wears is the standard way of signalling in sculpture that he was a stratagos, one of Athens’ elected generals. The artistic convention would seem to suggest that the strategoi tended to wear this older (and higher status) type of helmet for some time after it had become quite rare among regular hoplites, though of course we must admit the possibility that it is merely an artistic convention.

And especially for armies which are not built for or do not require a lot of careful handling once the fight starts, the best thing a general probably can do is put himself in a highly visible position. We’ll get into this more next time, but most pre-modern armies are essentially single-function machines anyway: you line the men up, hit ‘go’ and hope for the best. If your component shield-wall or shield-and-archer blocks can’t maneuver in combat anyway, the general might as well cut a dashing figure and at least do some good in the minds of his soldiers.

Via Wikipedia, a statue of the Roman emperor Trajan (r. 98-117) dressed as a general from Xanten, Germany. Sculptors enjoyed embelishing the details on the cuirasses of emperors, but this style of armor – the muscle cuirass – was essentially the province only of elites and particularly of commanders in Rome. Wearing such would have made an officer immediately recognizable.

But what if you do want to do some command without completely surrendering these leadership advantages?

The Battle Manager

That at last brings us back to Everett Wheeler’s concept of the ‘battle manager,’ a phrase Wheeler coins off of Plutarch’s description of Pyrrhus of Epirus commanding at the Battle of Heracleia in 280 (Plut. Pyrrh. 16.8). Plutarch stresses both that the battle “did not blur his power of calculation” but that Pyrrhus also was “running from one spot to another and bolstering those seeming to be overpowered” and thus “managed the battle.” Wheeler is mostly concerned with battle manager generals in the Greek context in that particular chapter, but I think we can extend the idea a little bit and draw distinctions between two models, which I’ll call the ‘Alexander’ model and the ‘Caesar’ model. Both sit between the warrior-hero-general and the pure-commander-general on the leadership <-> command spectrum, but the ‘Alexander’ model is a bit closer to the warrior-hero and the ‘Caesar’ model a bit closer to the ‘commander’ model.

Before we dive into either, it has to be noted that for a battle manager to work to its full potential, the army has to be organized correctly for it. In particular I hope you will note in the next few paragraphs how important subordinate and junior officers (some we might classify as NCOs) are to this system working, for two key reasons. First, the battle manager has to be able to delegate to these fellows. He cannot be everywhere and almost certainly has an army that is complex enough to require some coordination and command at all levels so he needs subordinates in command of each component unit who can handle well enough on their own in normal circumstances (and who can also perform visual leadership for their own smaller unit). At the same time, he has to be able to command through those subordinates, for the reasons we discussed above.

If we look at the way Alexander commanded in battle – even beginning with Chaeronea before he was king – there is a fairly common pattern. Command of the infantry bulk of the army – the Macedonian sarisa-phalanx – was entrusted to key subordinates. In this case the structure was that there was an overall commander of that section of the army (early on this was Parmenion), which was then broken into six taxeis, each commanded by a taxiarch who was a noble Macedonian companion of Alexander’s. Those taxeis were then subdivided into the basic unit of the phalanx, the syntagma of 256 men which seem to have been able to maneuver independently, but normally would be joined together to make the taxeis one coherent wall of sarisa-points. Other units similarly had commanders and sub-commanders.

Via Wikipedia, a map of the Battle of Gaugamela. Notice how Alexander is effectively in charge of his cavalry contingent and nothing else, trusting Parmenion to hold up the left flank long enough for Alexander to achieve a decisive result on the right.

Alexander himself rode at the front of his Companion cavalry, his elite striking cavalry formation. There’s a good deal of sense to that: Alexander is the fellow who comes up with the battle plan and is supposed to be thinking about the situation on the whole field. By being with his most powerful, mobile striking force, the moment he sees an opportunity, he can move immediately with that force to capitalize on it (alternately, other generals in the Alexander mode, if they saw a danger, could move with that force to stabilize it). And that’s exactly what Alexander does, over and over again: the pressure of the phalanx creates weaknesses in the enemy force and when Alexander sees those gaps, he leads the Companion cavalry (with elite light troops in tow to support) straight into that gap.

Also from Wikipedia, Alexander achieving a decisive result on the right.

Now two notes here: first what this means about Alexander’s command opportunities. He really is driving the Companion cavalry and relying mostly on his other commanders to handle their parts of this army. His ability to influence them – beyond coming up with the general plan before the battle – is limited. At best he could send a messenger, but obviously once he commits the Companion cavalry (and himself!) that’s hardly an option anymore. On the flip-side, by commanding the Companion cavalry as a sort of ‘reserve’ (albeit an offensive force he intends to commit fairly early) he still has some control over the course of the battle. The second thing, of course, is that he sacrifices very little of the leadership benefits in so doing because he is still charging forward, highly visible with his very distinctive helmet and cloak, at the head of the cavalry where everyone can see him being aggressive, confidence, involved in the battle and sharing in the peril of his men.

Via Wikipedia, a detail of the Alexander Mosaic from the House of the Faun in Pompeii, an early first century BC copy of a likely third century BC original, depicting the Battle of Issus. Alexander rides boldly at the head of his Companion Cavalry charging directly at Darius (out of picture here).

Caesar provides a different model, though it should be noted this isn’t particular to him but rather Caesar performs with uncommon excellence a broadly common Roman model of command. First, Caesar did not particularly attach himself to any unit in his army; he rode with his own retinue of course but this was not a dedicated striking force the way Alexander’s larger body of Companion cavalry was. Instead, reading his battles and campaigns Caesar seems frequently in motion or else positioned on ground designed for observation. At Alesia (52BC), for instance, Caesar expressly notes that in the chaotic final engagement he “found a suitable place where he could see what was happening in each part [of the battlefield]” (Caes. B.G. 7.85) and from there dispatches his reserves as necessary to repel the attacks, sending subordinates (Labienus, Brutus and Gaius Fabius) with chunks of his reserves at various points to key distressed places.

What I want to note here is that Caesar can only send the reserves he has. Even being Gaius Julius Caesar with all of the skill that implies, the moment he sends these fellows he loses control of them. Once a cohort – Caesar’s battlefield maneuver unit of preference, representing 1/10th of a legion – is engaged, Caesar can’t do much to control it anymore and rarely tries. What he does do is move around unengaged cohorts, which conveniently for him, the Roman triplex acies and the generally large size of his armies tend to create.

That doesn’t mean that Caesar never gets ‘stuck in’ to perform the warrior-hero role, though. Rather, like most Roman generals he only does so when the situation seems critical and the morale effect of seeing the general himself charge into combat is most needed. After sending Labienus to bail out a weakness in his position at Alesia, Caesar himself goes to the point to encourage his troops, though he doesn’t yet engage (Caes., B.G. 7.86), in part because he seems to be waiting to see if Brutus and Fabius can resolve the other critical point in his line; when they do (7.87) goes to where Labienus is still pressed, “rushing that he might get stuck in the fight.”14 The intended impact here is moral as Caesar is quick to note that he was readily visible to all by wearing the general’s distinctive red cloak.

At the same time, it would be a mistake to attribute all of the flexibility of Roman legions to their commanding generals. To take another example, at Cynocephelae (197BC), a fight between the Romans and the Macedonians, the Roman left had been pushed back while the Roman right had pushed forward (the Macedonians opposite them were still forming up and hadn’t pressed their position), creating a gap between the two sides of the battlefield.15 The Roman commander, Flamininus, seeing his left losing slowly seems to have gambled he could win on the right quickly rushed there and led a strong attack (with elephants!). Flamininus, for his part, seems to have lost control of the battle mostly here; most of his troops pursued the fleeing Macedonians on the right with the potential danger that they might chase them off of the field, allowing the Macedonians to win on the other side of the battle. But at this point an unnamed junior officer, a military tribune, rallied up twenty maniples (about half a legion) and turned and plowed them straight into the Macedonian flank, winning the battle. Junior officers like this frequently matter in Roman battles. To take another example, Caesar’s army at Bibracte (58BC) famously manages the feat of winning when attacked in both the front and the rear by about-facing its third line and attacking in both directions at once. Notably Caesar does not say that he did this, merely that the Romans did, which is a strong indication that turning the rear ranks of the army to face backwards was a decision made by more junior officers (probably centurions). Caesar for his part was, somewhat rarely, unmounted at this juncture and so perhaps not fully aware of the situation (he had sent away the horses as a heroic gesture to share his soldier’s peril as a way of firming up morale in an army that was, at this point, larely untested).

Via Wikipedia, a rough map of the Battle of Cynocephelae (197).

Towards a Model of Army Command

To sum up then we might ask again how much control does a pre-modern general have over his army once the pitched battle has started? The Total War vision presents an almost infinite canvas of possibilities for the general to make many, many decisions. By contrast actual generals, constrained both by cultural expectations and the simple physics of battlefield communications without radios, telegraphs or cellphones, really had only a handful of ‘decision points’ where they could effect the battle.

Prior to the battle the general is going to come up with the battle plan: how the army will be arrayed, how it intends to fight and how he thinks it will win. We’ll come back to this next time but in many cases this too is a socially-embedded, culturally contingent ‘script’ – especially in unplanned battles where the ‘ideal’ battle everyone has in their head is the only plan anyone has to work with. In any case at best most of the army is going to try to execute on the plan as laid out and the general is going to have very little ability to change that both because of limited communications and, as discussed last time, limited information. That means the broad outlines of the plan are set the night before the battle, with some knowledge of the terrain but no knowledge of the enemy’s battleplan save for informed guesses.

As the army forms up the general has to decide where to be. In some cases, this is going to be culturally embedded and thus hardly a choice. Macedonian kings – the Ptolemies notwithstanding – ride with their elite cavalry (the Companions or later the agema) and it would be strange for the king to be anywhere else. Spartan kings deploy on the right flank of their army (though not always, it seems, in the front rank). For other armies, this will be a choice. Flamininus, as noted, chooses to rush to his victorious right rather than his beleaguered left in the hopes of pulling out a victory. Last week, you will recall Aemilius Paullus makes a clear decision to ‘drive’ his Legio I in the battle (so taking his beleaguered right over his safer left). The general’s control over the unit he is with is going to be much higher than anywhere else so this decision, “where should I be?” shapes the decision-points that come after it.

Once the army is advancing, the general may, if his form of army allows for it, have a key decision about when and where to commit his reserve, which is typically going to be either the unit he is with, or part of it. However, many pre-modern armies have little in the way of a reserve – reserves demand a command-oriented (at least ‘battle manager’) general to deploy them and the command structure to allow them to be deployed. And of course forces pulled into the reserve are not in the main force and there is always a risk there: one thinks of the Macedonian cavalry reserve at Pydna (168) which never deploys (the battle is lost before they are committed) and it is hard not to conclude they’d have been better used as part of the main force.

Finally, as the fight proper begins, the general has a crucial choice about where to commit himself. In many pre-modern armies, this won’t be much of a choice at all, because the general’s position is preset, but for more flexible command systems this choice is crucial. By committing himself to a specific point, the general can produce local morale effects or exert much finer control over a very specific unit (like the Companion cavalry), but doing so effectively takes him out of the rest of the battle, meaning that everything else about the plan now relies entirely on his subordinates. It is striking that Caesar tends to wait until things are truly critical before ever engaging himself; only when he has dispatched all of his reserves does he consider dispatching himself (which makes sense: once he charges in, he’s no longer in a position to order his reserves to do anything anyway).

And that is about the limit of the command options available to the general; beyond these basic decisions the general is mostly stuck hoping his subordinates and the rest of the army can execute on the plan. Needless to say it is not hard to see why mainstream tactics games do not work like this: having to make a plan effectively blind and then mostly just hope it goes well would be very frustrating for many players. It was frustrating for many generals too, of course!

Next week, we’re going to complicate this picture even more by looking at the army itself and how its organization, subordinate officers, level of drill and informal doctrine further shape and constrain command decisions, but also situations in which they allow armies to respond dynamically without the general’s input.

  1. In fairness, ‘rush’ may not be an accurate word to use to describe Creative Assembly’s currently glacial pace of development on TW: Warhammer III, though my sense is that this was a resource allocation issue within CA and no fault of the developers themselves.
  2. I feel the need to note that Braveheart is one of those films that mangles its history so badly that I can’t really watch it. The characters are wrong, the battles are wrong, the equipment is wrong, the sets are wrong. Only the soundtrack is good and mercifully one can listen to that without having to experience the rest of the film.
  3. Ultimate General: Gettysburg and its sequel, Ultimate General: Civil War attempt, to a degree, to actually give their colonels and brigadier generals some independence and so they occasionally will not advance when you tell them to, or retreat when you tell them to hold, if morale is low. But you never glance over to your left flank to find out that Dan Sickles is just not there because he has decided to position himself 1.1 kilometers in advance of his intended position in an act of pure insubordination. Not that I am saying that would be fun gameplay – but it would be accurate gameplay.
  4. In actual warfare, doing something like that would often cause the line to break before contact because your soldiers are not stupid and if you charge them into something obviously suicidal, they’ll just run away – they won’t typically wait to take 50% casualties first.
  5. The description that follows borrows a fair bit, including the term ‘battle manager,’ from E. Wheeler, “The General As Hoplite” in Hoplites: The Classical Greek Battle Experience (1993).
  6. Of course these are fictional works, though the communis opinio of scholars is that warfare in Homer is mostly a pastiche of warfare in his own day (that is the late Dark Age/very early Archaic) including the preference for commanders to ‘lead from the front.’
  7. Though where the warrior-hero general is also the king, state accounts of the battle may choose to represent it as if the king slew the enemy all on his own, as with, for instance, the Kadesh Inscription.
  8. This episode is often presented with the rose-tint of Lost Cause framing as showing Lee as beloved of his soldiers, but it should be noted that in this same battle, Lee’s best living corps commander, Longstreet, also proceeded recklessly forward and was wounded by friendly fire, not very far from the spot where Jackson had been killed by friendly fire two years earlier. So one my well instead read it as a sterling example of how the ‘chivalrous’ (their word, not mine) ideal of leading from the front held by the enslaver-planter aristocracy of the South was frankly unsuited to the warfare of their era and actually detrimental to their battlefield performance.
  9. Technical term meaning ‘close combat’ as with swords, spears, bayonets or other ‘contact’ weapons.
  10. A summary of this research is presented early on in E. Anson, “The General’s Pre-Battle Exhortation in Graeco-Roman Warfare” Greece & Rome 57.2 (2010): 304-318. The question of the ability of a speaker to speak to a large crowd is also a central one in debates about the nature of the Roman Republic, e.g. F. Millar, The Crowd in Rome in the Late Republic (1998), H. Mouritsen, Plebs and Politics in the Late Roman Republic (2001) and R. Morstein-Marx, Mass Oratory and Political Power in the Late Roman Republic (2004).
  11. There is a necessary caveat here that Vegetius is a difficult source. Writing in the fourth century AD he imagines himself describing the legion of the first or second century AD, but indiscriminately mixes source material from as early as the second century BC with his own time.
  12. This is actually something simulated in the Take Command series: orders to distant units do have to be sent by messengers and if that messenger gets shot then you must send another.
  13. That isn’t to say leadership in modern armies isn’t important – it is very important, potentially more important because it cannot usually be done visually.
  14. Accelerat Caesar, ut proelio intersit, literally: “Caesar hurried so that he might get into the middle of the combat.”
  15. This whole battle narrative is in Plb. 18.24-26

220 thoughts on “Collections: Total Generalship: Commanding Pre-Modern Armies, Part II: Commands

  1. I think it’s interesting that you compare the player of a game to the general on the field in that game. That’s not really the mindset I have playing them. Maybe it’s something encouraged by the presentation of Total War; I don’t have any experience playing it. But for example I don’t think of myself as Tassadar in StarCraft. He’s just one character of many. I’m The Protoss, collectively. I’m not roleplaying Hammurabi in Civ, I’m playing as the Babylonian civilization. I can command any unit to do whatever l want it to do like I can wiggle my big toe because I am playing that unit as much as any other. Of course playing as a sapient, disembodied collective will is even more alien than a mere omnipresent general would be, but that’s videogaming for you.

    1. Starcraft does canonically define the commander as a character, even if they’re the typical anonymous avatar with no character beyond “follows the main character around and fights battles for them.” In Starcraft you’re an unnamed Colonial Magistrate, an unnamed Zerg Cerebrate, and an unnamed Protoss Executor. Starcraft 2 goes even further by retconning the commander characters out of existence – you play directly as Jim Raynor, Kerrigan, and Artanis.

      It’s obviously heavily abstracted – Artanis will say “Aiur is before you, march forwards and take it!” and then the army will wait patiently for the player to issue the “Attack” command before they actually start moving – but in most RTSes there’s an actual character in charge of the army and the player’s interface is just an abstraction for the commands the character is giving.

      (The Zerg and the Protoss have the justification of being literally psychic, but the Terrans seem to get by with nothing but some fancy computers. We see Jim Raynor carrying around a little computer-briefcase-projector thing in the missions where he’s away from the Hyperion.)

      1. You are totally right as far as the story goes. Same is true in Command and Conquer where the characters will chew the scenery for a bit then talk directly into the camera, meaning directly at the player, and tell you what to do. In Supreme Commander you even get represented on the battlefield as a giant mech. But in terms of how it “feels” during gameplay it’s never “I’m issuing orders to these guys over here” it’s “I move over here and attack”. I’m sure it’s largely a consequence of the complete lack of friction between decisions and execution as outlined in the article.

        There are games that don’t make me feel that way, though. For example in Sim City I’m not the city, I’m some sort of autocratic tyrant-mayor. In Tropico I’m explicitly a dictator and the game makes you feel it through a lot of presentation, even though you still get a lot of the unrealistic level of command and control Bret discusses.

        1. Hmm… perhaps the abstractions of “you’re a specific general” or “you’re the army as a whole” both disguise the underlying fact that these are fundamentally puzzle (and reflexes) games where the goal (at least in single-player campaigns) is to figure out what steps you need to take in order to defeat the enemy. In almost all cases, the outcome of the battle (in terms of the story) is predetermined; your side wins, otherwise you don’t get to proceed in the story and have to try again, making it as if the first attempt never happened. (Or in some cases, your side “loses” but you still completed the puzzle and the story proceeds.)

          To that end, these abstractions affect the degree to which you are actually simulating realistic warfare, and that degree even in the most realistic games is effectively zero. The art and sound assets make it resemble warfare, but almost nothing in the game represents real warfare of any kind at any point in history. And that means that while these games can certainly be fun and challenging, and can perhaps make one interested in learning about real history (military and otherwise), the media themselves do not teach *anything* about history, except occasionally by accident.

      2. Not to nitpick, but I thought Artanis is the player-character from the original Protoss campaign, and got promoted at the beginning of Brood War to fill Tassadar’s shoes, no? Interesting to consider that at least in the original game and expansion, all three factions (even the two with some form of psychic hivemind power) seem to operate on a system where the role of the figure in charge is to do Alexander-style leadership from the front, while the one taking advantage of all that futuristic communications technology and/or telepathy to do modern battlefield commander stuff (i.e. the player-character) is a subordinate officer who’s cut out from any and all bigger-picture strategic decisions aside from the occasional “do we take out their battlecruisers first or their nukes?”

        1. Definitely not – the player character in Brood War is the same unnamed Executor as in the original game’s Protoss campaign. The Terrans are the only faction to have a change in player character between the original game and the expansion, since they represent an entirely different faction.

          1. https://en.wikipedia.org/wiki/Characters_of_StarCraft#Artanis

            Artanis is a high templar and a military commander introduced in Brood War. Later retcons […] have established Artanis as the identity of the nameless ‘Executor’ for the Protoss campaign in the original StarCraft.

            I’d also been under the impression that the Terran player-character from the original game was supposed to be Matt Horner from the sequel (Horner’s role as the dude who stays on the command ship and does communications/support/logistics/etc while Raynor leads the troops on the ground seems pretty close to the role of one of the original game’s player-characters) but apparently that’s not the case and the canon narrative is that the former Mar Sara magistrate just decided to up and quit for some reason.

          2. Blizzard just wrote the original player characters out of the narrative and compressed everything to having been done by Raynor, Artanis, or Kerrigan. The Magistrate and the Cerebrate have been retconned out of existence.

        2. Player character was originally a separate person, but Blizzard changed the story and created a messy character mashup in this case.

          Selendis as the brood war executor is a proposal I’ve seen, which works fine.

          1. Selendis’ character makes some of the more… interesting strategic decisions in the Protoss BW campaign make sense, too.

          2. It’s telling that the in-story player characters of the Starcraft and Brood War campaigns are such non-entities that there’s serious dispute over who they even are.

    2. Ok this is rather off topic but this does make me wonder. If this sort of weird ”sapient, disembodied collective will” intelligence existed in real life how would warfare against it go compared against fightning armies of regular humans?

  2. Typo:

    1/0th of a legion –>. 1/10th. (infinite legions would indeed make an army powerful, but would have logistics issues I imagine.)

        1. Ending up in another universe does not help when you have been torn apart by tidal forces.

        1. Seriously, that’s why the idea of “nothing” as a number took so long to adopt, in part precisely because of such conceptual difficulties.

  3. More comments on a Dominions type “give orders and the computer acts them out for you” system. For fun, it could have advantages of:

    -Saving time, you don’t need to fight every battle, or rely on often wierd autoresolve systems, just set your army up and watch or not if wanted.

    -Designed well, it still tests skills and gives you something to learn:. Did you set up a good plan?

    -I could see a combined method where prepared orders could be changed if the player wanted under some conditions, but this sort of combat likely would limit micromanagement which avoids some “must click several times a second” intimidation”

    Another system I’ve thought of is only allowing a certain amount of orders per time to each unit, depending on some leadership measure, meant to represent the “command hive mind” as a comment last week put it, of how good subordinates are, while also cutting a clickfest, and giving a tradeoff between a larger army (that you’d position well) vs. a smaller, better controlled one.

    Though all this is about making fun games, not how leadership actually worked. Article is a good read for that.

    1. I was just thinking that such a game could do very well if made and sold right. It also reminds me somewhat of a variation of chess I’ve played over at chess.com.

      There you and your oppponent take turns setting up your pieces (you have a points limit, rather than just a full set), within certain requirements (minimum six pawns, one King, must deploy those six pawns first) and then the computer takes over and actually plays the game for you. It’s quite interesting.

    2. Sadly, dominions is NOT designed well and its order system is hugely opaque and difficult to input, and midgame what your troops do doesn’t matter, it’s all about how you script your casting.

      I think the majesty games capture this feel pretty well though.

    3. It seems weird to describe the Dominion scripting system as saving time. Trying to program a big battle or a lot of little ones could a lot of work. I’m pretty sure I’ve spent more time on single Dom4 turns than on any TW one. Certain actions, like trying to buff specific guys or make units of different speeds hit make contact at the same time could be quite finicky to arrange.

    4. The old Slytherine games, Legion and Spartan, were somewhat like this (and they were a ton of fun, by the by – they might still be available on iPads, I should check sometime). You would position your forces in a battle with formations and simple instructions (advance, hold, or wait for a short/medium/long time before advancing, etc.), and then the battle would start and you’d really just have to watch it play out.

      It honestly was quite intense at times, helplessly watching a close battle without any real ability to make further commands – except an order to retreat, in which case your army would rout and you’d have to hope that a good chunk of its units would still exist post-battle.

    5. “Another system I’ve thought of is only allowing a certain amount of orders per time to each unit, depending on some leadership measure, meant to represent the “command hive mind” as a comment last week put it, of how good subordinates are, while also cutting a clickfest, and giving a tradeoff between a larger army (that you’d position well) vs. a smaller, better controlled one.”

      There’s a game in early access on Steam called Winter Falling: Battle Tactics that has an interesting take on time-limited orders; you have a limited number of commands (represented as birds) with separate cooldowns that you have to use to issue commands, either box-selecting units and moving them as a mass, or giving more specific orders to a single unit. It definitely allows way more control than actual battles would have, but it’s an interesting system.

  4. What was the calculus like for the possibility of a general dying because he’s on the front line. Even if he can’t do anything more to control this battle you may want him around for the next one. And if showing bravery at the front improves morale, what does visibility dying do? I imagine different cultures and different individual people react differently in the even, but do we have many records of this?

    1. Oh, the general dying (or being falsely reported dead) is an absolute morale catastrophe. At Hastings, the Normans routed when William was reported dead. Then the Saxons pursued, and William, being alive, managed to rally his troops and counter-attack to win the battle.

      1. It’s going to depend on the culture and composition of the army and the context of the battle as much as anything, I suspect. Hastings is an interesting example. What we have there is an army of Norman adventurers and mercenaries organised in largely peak-feudal manner, fighting a nominal crusade for the ambitions and on the reputation of a single man. William’s survival was critical to the entire venture and if he fell in battle not only was the entire enterprise pointless but it didn’t speak well for divine judgement on the venture at large.

        For the English, by contrast, in some ways a more “professional” army at its core, fighting off an invasion by raiders threatening to burn their homes etc., Harold’s death in isolation might not have been so critical: the troops had an independent motivation from the person of their commander, there were other established, capable war-leaders on the field and Harold’s survival was not in itself critical to the kingdom provided the battle could be won (Harold had after all been elected largely in order to defeat the invasions). The crown could be passed without too much fuss to one of Harold’s brothers or to Edgar (as indeed it was). Morale would doubtless take a hit on the loss of the king but the English army was probably set up to resist that shock better than the Normans were.

        While it’s not wholly clear exactly what happened, it appears that Harold was wounded almost incidentally to the overall progress of the battle and then killed once it had already been effectively lost. The kicker was that Gyrth and Leofwine were killed too, leaving the English army largely leaderless in the south. Along with them were lost such a large proportion of the remaining English elite soldiery that the key English power-brokers lost confidence in their ability to sustain the defensive campaign, so even though there was an initial rallying to Edgar, many of them quickly made independent peaces with William.

      2. The general dying, and someone as a ruse dressing up like them and pretending to be still alive is a common *fictional* element, but it shows up a few times in allegedly historical sources too.

        I do note that at Lützen Gustavus Adolphus death AND the fact that the battle wasn’t lost despite it probably both depending on the simple fact that it was foggy, and no one could see where he was.

        1. Now that makes me wonder: in conditions of miserable visibility, could an army of disorganized melee fighters defeat one that normally was better due to formations and coordination?

          1. You just described the battle of Teutoburg Forest. Romans vs. Germans, at night, in a storm, in a forest.

    2. Depending on the culture and the battle, a general who loses a fight may die anyway. So if fighting on the front lines brings victory, it makes sense to do so, even if there is extreme personal risk involved.

    3. I think the answer is that, generally speaking, losing is costly enough that it doesn’t make sense to plan for it as a contingency. Losing a battle means the winners chase you down and kill as many of you as possible, then raid your camp to take all your supplies. Under the circumstances, planning for the next battle is foolish, because there often won’t be a next battle.

      1. Yes, but the general visibly dying on the front lines might be worse for morale than having him standing around in the back. And also it’s just rarely clear that the general personally leading from the front will make the difference between victory and defeat.

        1. Mostly losing a leader is so bad for morale that side collapses. Also, in medieval warfare. the fights were often about who is to be leader. Losing yours by death or capture ends the fight – there’s no point going on. Sometimes (Alexander, Byrhtnoth, Gustavus Adolphus) the death (or news of death) arouses a fury, and the troops fight harder – sometimes even winning.

          1. This is a late response, but in Jordanes’ description of the Catalaunian Fields the death of Theodoric seems to make the Visigoths more spirited, with them charging and nearly killing Attila

        2. Depends on how visible it is. One more dead body in a mass where you can’t see much anyway because of your helmet. . . .

          1. Remember that generals leading from the front try to be as visible as possible.

        3. Yes, but if the general isn’t already dead…

          That argument cuts both ways.

          The general visibly standing around in the back on a horse in a great position to gallop away if things go south might be worse for morale than having him standing there in the front lines sharing the danger and expressing confidence in his own battle plan and his own troops. And also it’s just rarely clear that the general being in a position to dispatch parts of his limited reserve force, while working around the limits of the available communications, will make the difference between victory and defeat.

          Everything here is an uncertainty.

          If the general fights on the front lines, he might be killed. Then again, he might not. After all, he’s wearing good armor and his personal guards will fight bravely to protect him. His odds aren’t all that bad, really, especially if he doesn’t outright push forward to the very front of the battle and is just, y’know, there. It’s not like both sides have a ton of snipers with high-powered marksman rifles lying around to pick off anyone wearing shiny armor who gets near the front.

          Meanwhile, if the general stays back, that risk of him being killed is omitted, but the risk of him being ignored, or of his army (which is used to and raised on stories of heroic leadership) deciding that he’s afraid or lacks confidence, are all real dangers that could cost him the battle and potentially his life.

          1. @Simon_Jester

            Seems that Asian Armies historically are quite biased towards Generals not fighting in the Front line. But Commanding in the middle of his army or from the back

        4. And also it’s just rarely clear that the general personally leading from the front will make the difference between victory and defeat.

          If your soldiers expect you to fight heroically from the front, it seems very plausible that your instead choosing to skulk around the rear like a coward would demoralise your soldiers and cost you the battle.

    4. Well if nothing else, the general has the authority to order a retreat if things are looking bad. 🙂

      1. In practice, by the time a commander far back from the lines is able to order a retreat under these conditions, the troops actually present on the field are already starting to fall back anyway.

        Unless the general has a strong reserve force to cover the retreat (unlikely, since if he had it, he’d probably have committed it earlier to prevent the defeat that is now happening)… Well, there’s not a lot he can do from the rear that he couldn’t do from the front.

        1. We were talking about the perils of a general fighting at the front. I tongue-in-cheek suggested that he would be the one to order a fall back if the front was truly in danger.

    5. The general was not usually in the front rank. He’s also fiercely defended. And in early to mid medieval times, the personal household of the leader are likely the strongest force on his side, and a significant part of the whole force. Your chances of winning are greatest if they are committed.

  5. Just to say I enjoy this series so much! Also, kudos for mentioning here – although this could be emphasized a bit more and not relegated to a footnote – that video games ultimately have to make a difficult trade-off between realism and gameplay, with most of them sacrificing the former for the benefit of the latter, for obvious reasons.

    In fact, I can remember, in one of my campaigns of the original Rome TW, I had a general which was fairly competent on the surface (4 or 5 stars out of 10, in game terms), but also had a number of traits (which I initially overlooked) that had a crippling effect on the troops morale. That lead to the situations that my troops actually WERE breaking even before contact in some more precarious situations (sieges being the prime example)…which I distinctly remember being frustrating AF!

    I also tried a few times to play various TW games in a more realistic manner (manual battles only with my faction leader as my “avatar”, locked camera, no pausing which, in concert with my limited micro skills, lead effectively to a near-to-real lack of control). And in the olden days of one of my old PCs, playing RTW was so laggy that I often joked about having about as much control as a real commander – orders being implemented with massive delay or not at all, me seeing what happened a few seconds after it actually happened… But, as the game was not really built for any of this, it lead mostly to frustrating moments and nothing else. So, realism does take a lot of fun from works that are primarily meant to be entertainment.

    That said, I think I’d like to see a game where these kind of things are done as realistically as possible – like delayed and limited communication, giving only limited orders, even higher importance of morale (although I must note, this last point is done better in some older TW games than Bret gives them credit for; not ideal, but better) etc. Can’t know if you’d like it (or if it’d sell) before you try it…

    1. The success of the Football Manager series suggests that there’s plenty of space for a game that’s about picking your forces and tactics and then leaves the actual contest mostly beyond your control except through either sending on substitutes (reinforcements) or attempting to change the tactics mid-game.

      1. Good point! ‘Course, the fact that football is a bit more popular than pre-modern battle simulation also helps 🙂

        But joking aside, it is a good example that even a game consisting of not much more than text and colourful geometric shapes moving around (or not) can be extremely successful. So we can be hopeful…

    2. Imagine a conquest of alexander the great simulator. You draw up a formation beforehand (usually the standard one), then the battle starts with you in first person view of alexander. All you can do is wait and charge.

      1. The game “Middle Earth, Shadow of War” KIND of does this. When assaulting a fort, first you pick the captains who will lead the assault (based on their attributes) and pick what extra perks they get based on the defenses you face. Once you commence the attack, you have to trust that your forces will do their thing while you yourself fight in the standard third-person action rpg way and back them up. I think a game with a more elaborate battle plan phase, but having you still fight for yourself when the battle begins, could work pretty well.

      2. Perhaps the best way would be to do a sort of hybrid system: you play as the general, and can order around the individual units within a certain radius of your position, Total War style; for units beyond that radius, you set the overall priorities before the battle, and then the AI manages them for you.

        1. Outside of the regular thrust of the historical discussion but there are three levels of tactical control in the (sorta niche) naval game “Rule the Waves”. It’s naval combat 1900-1950 (roughly) and in the tactical battles you have the choice of level of command control in exchange for a battle VP penalty:
          1. Captain’s Mode – you can issue direct orders to all ship divisions. Also allows manual firing of torpedoes.
          2. Rear Admiral’s Mode (default) – you can issue direct orders to all ship divisions *in visual range* of the fleet flagship. Night actions can make this very interesting/frustrating.
          3. Admiral’s Mode – you only issue direct orders to the ship division the fleet flagship is in.

          The various divisions can be given orders regarding formation, role, who to follow, etc., but the AI is interpreting those orders. (Often poorly or slowly.)

        2. I’ve love to do something like this, with the addition that picking your generals changes the AI used. One might be more aggressive, one overly cautious. One follows your orders perfectly, another might inspire the troops better but may line up 1km ahead of where your flank is supposed to be just because he felt like it.

          You play your section of battle, they play yours, all in real time. Maybe you can send messages by horseback to update their instructions or to reinforce them. I think you could make it fun.

      3. It doesen’t *quite* work like that, but the original Rainbow Six games (where you play a SWAT team/counterrorist force) has you first draw up plans for your AI squadmates, and then take control of one of them in first person. (including stuff like “Wait at point Bravo until I give the signal to execute the next step of the plan”)

        It wasn’t quite that restrictive since you could magically switch between different squad members, but it was pretty close.

    3. I think a fun way to have a battle where the player has limited control be fun is to go all in on the Hero General. The player controls the general and your general has only limited ability to order around the army but an excellent set of ability to kill dudes personally. So what the player would largely be doing is controlling the lead from the front general hacking his way through enemy formations, trying to find the enemy general and kill him personally, etc.

      Meanwhile around you the battle rages but you’d have only a limited ability to micromanage it mostly having things like a morale boosting aura and being able to issue “hey dudes that can hear me, FOLLOW ME.”

      1. That’s basically the Dynasty Warriors series, except that Dynasty Warriors does have a mini-map (otherwise you’d spend the whole battle just trying to figure out where to go). There’s usually no ability to give orders at all (even though you’re always an “officer”), and morale effects are entirely from officers getting defeated or scripted events.

        1. I think Kingdom Under Fire and Koei’s own Bladestorm: The Hundred Years War are both interesting case studies. Both games clearly lean heavily towards the general-as-warrior-hero model seen in Dynasty Warriors, but they also allow you to take command of a unit and “drive” it. So that’s an interesting attempt at creating a “battle manager” game.

          1. There’s a kinda janky Dynasty-warriors esque MP game called Tiger Knight Empire War (as you can read the translation is… a thing) where you basically have command of a unit of people who follow you around, and you can give them simple commands, but also have to do the fighting and hacking and such.

      2. How would one organize an army around a single invincible super-warrior? One who is effectively invulnerable and can smash any enemy in his personal shock range, but who cannot be everywhere at once, which wouldn’t help the rest of his army if they all got wiped out. What would be their most effective use?

        1. I figure you’d want to avoid the big field battles, then.

          Have the superwarrior break into the enemy camp, then have cavalry positioned to chase them down as they flee.

          Or hit them when they’re stretched out along a road. The superwarrior starts at one end and keeps going until the enemy scatters.

        2. Depending on how fast they are you either stick them directly opposite the absolute toughest part of the enemy line to smash a hole in it, or have them run back and forth to anywhere that looks like it’s in trouble.

        3. Well if they are truly invincible and invulnerable they’re basically a god. So it’s not how *you* can use *them* most effectively but more how can you convince them that you are a loyal subject while those other guys are not.

          For the merely super skilled warriors of fantasy and legend, follow the model of Alexander the Great and use them to break the enemy line. If we assume the stories about Viking berserkers are not entirely fanciful that does seem to have been how they were used. Vikings and most of the people they fought relied on a shield wall for mutual protection rather than good individual armour. A few warriors who genuinely did not care if they died in the process could create a breakthrough to be exploited by the rest of the army.

          In an era where generalship is more about displaying bravery and fighting in the front line, point the super warrior at the enemy general. This happens in the one Conan story (that I remember) where he’s the general of a large army, A Witch Shall be Born. Conan is a cunning general (Fremen Mirage! Drink?) but this is entirely *before* the battle, drawing the enemy out from their fortified city to fight his army while concealing his newly acquired force of heavy shock cavalry. In the battle, Conan unleashes the cavalry and personally kills the enemy general.

          1. Despite being called the Barbarian, nothing is particularly Fremen about Conan. While he himself is a brilliant warrior and general, I don’t believe much is ever mentioned of the rest of the Cimmerians being particularly so.

          2. No other Cimmerians appear in the stories.

            His physical abilities, at least, are tied to his barbarian heritage, though I don’t know how much of that is Fremen superiority vs. racial superiority.

          3. In fact, it is explicitly mentioned that he cleared out because he didn’t like the culture.

          4. Well, yes, but Conan’s objection is that Cimmeria is a gloomy land full of gloomy people, while Conan himself is cheerful and talkative. It doesn’t really have anything to do with being Fremen.

          5. Conan’s physical abilities are often ascribed to his barbarian heritage, which I think is meant more racially.

            However, he picks up generalship and especially kingship as he goes; he often complains about how kingship is a lot more difficult than becoming king. He ends up being a solid one and a good general, both in performing leadership and tactics. In The Hour Of The Dragon, the enemy wizard preemptively disables him because he wouldn’t fall for the trap, and when his replacement (disguised as him) does fall for it Conan immediately senses it just from his page’s description.

          6. I find the stories where he ascends to leadership roles unconvincing. In one he’s appointed by an oracle; in another, he kills a pirate chief and becomes a chief of a pirate band that never met him before; in a third, we are told (but not shown) that he wins a crew over.

  6. Have you ever tried Scourge of war: Waterloo? It can be played similar to a total war battle (with full freedom of camera movement, micromanaging individual units), but it has a mode where you can limit yourself to only being able to see what your general is able to see on horseback (camera is still a little elevated to the 3rd person view), and more interestingly, you can play it solely by giving general orders to your subordinate commanders. For example, if you play as napoleon, you can limit yourself to only giving orders to your corps commanders (for example, advance and attack on the left in 2 hours time). Wonder what your view is on whether that game is able to recreate most of the feel of commanding a Napoleonic army.

  7. Strangely enough, so far the game I’ve played that seems most like an accurate simulation based on these is descriptions is *Hyborian War*, an old play-by-mail game by Reality Simulations. Every month you would get a packet describing the status of your kingdom and a sheet to mail in with your orders for the next turn. When one of your armies bumped into another army (I don’t remember how scouting worked – maybe I just didn’t do a good job at it!) there would be a battle the next turn – the type of terrain would determine a battlefield width, and your orders for that turn would include a grid to place your units. You’d fill this out and send it in, and the month afterwards you would find out how the battle went. (As I recall, you could set standing orders to try to give/receive battle in a particular type of terrain, and the approximate level of losses to accept before retreating, and that was about it.)

  8. Hooray! Early enough that I make some of the spelling corrections.

    “shiny metal statue mounted on a poll so it could be easily seen.”

    Presumably meant to be “mounted on a pole.”

    “at the head of the cavalry where everyone can see him being aggressive, confidence, involved in the battle”

    Meant to be “aggressive, confident, involved.”

  9. That mosaic of Alexander intrigues me, given how of then you have made a comment about ‘put on a damn helmet!’ in your writings on TV and movie depictions of battles!

    1. Artwork is there to honour a commander. Consider that pericles is not really wearing his helmet in that statue. Movies and TV however are trying to represent an actual battle. That is, unlike artwork, the characters aren’t supposed to be playing to a crowd. They shouldn’t have a need to show their face, certainly not enough of a need to not wear armour!

      Art created after a battle though wants the key players to be easily recognizable, so that the people looking at it can go “gosh our king/general/leader sure did win that battle”.

      1. “Movies and TV however are trying to represent an actual battle.”

        I wonder. Movies and TV are art and entertainment (if there is a difference between the two), not a documentary. In our culture, some amount of “verismo” is expected and often vocally requested, but maybe our revealed preference is less realism than often acknowledged.

        1. Oh absolutely. If tv and film was historically accurate it would make for a poor story. Hence why actors don’t wear helmets, canonballs explode, and everyone uses fire for everything.

        2. My impression is not so much a preference for realism, but for media to be consistent with the viewer’s idea of realism. Which is to say, consistent with the other media the viewer has consumed and liked.

          As an example, I watched The Northman with some friends. I really liked it, and – not being an expert on Vikings, mind – thought it gave off a fairly authentic feel. However, several of my friends complained that it was clearly unrealistic because everyone were so clean and had too nice teeth. Their expectation of a realistic Viking text is one in which everyone are covered in mud and their teeth are rotting. This is nonsense, of course, as people “back then” liked being dirty as little as us, and the in particular Vikings were famous for their cleanliness.

          I think that a lot of the time when we are asking for realism, we are actually shouting from the top of Mount Stupid, regurgitating Victorian stereotypes moreso than asking for consistency with modern historical knowledge, let alone actual realism. But, being on Mount Stupid, we don’t realise this. And so we watch shows like Vikings and feel it’s pretty spot on that they look like dirty bikers, because these were some rough and tough barbarians.

          1. Teeth is actually interesting, because while people might have bad teeth, they’d be bad in a different way than ours: Less cavities from sugar, and more ground-to-stumps from eatinv coarse-ground grain.

          2. Well there was a viking known as Wulf the Unwashed, but he would scarcely have been given that nickname if poor personal hygiene was the norm.

  10. Concerning footnote 7, I wonder how much of that is just part of a broader overall conception as to how leaders thought and talked in general. Consider the following passage from Deuteronomy, chapter 2, verses 26-28. Translation mine. For context, Moses is recounting the journey they took to get to the banks of the Jordan river.

    “And I sent messengers from the wilderness of Kedaimoth to Sichon, king of Cheshbon, with words of peace, saying: ‘Let me pass through your land. I will go along the road, I will not turn to the left nor the right. You will sell me food for silver, that I may eat, and sell me water for silver, that I may drink, only let me pass over with my feet”.

    All the same notes about actual historicity that our good host puts in about things like the Iliad apply here as well; I’m using this not so much as a claim to an actual interaction between a migrating Israelite group and a king in what’s now Jordan so much as a notion in that place and time of the ancient near east as to how kings (or people in position that are very much like kings) interact.

    But Moses’s demand (and I want to stress this is very much an ultimatum. I tried to translate with that in mind, but it’s far clearer in the Masoretic Hebrew than I can easily make it in English) is very clearly for some “we”. It doesn’t mean much if Moses personally is staying on the road or paying silver for the supplies he’s demanding, he clearly means it to apply to his people as well. But that’s not how it’s said; a leader’s following gets lumped in with the leader in this sort of formal Hebrew, even in a non-battle situation. I don’t know if the same linguistic conventions applied in Egyptian, especially much older Egyptian, since there’s a gap of centuries between the likely composition of the Kadesh Inscriptions and the book of Deuteronomy, but it wouldn’t entirely surprise me.

    1. Here Moses is using the figure of speech metonymy (using one word or name to refer to another thing), specifically its sub-type synecdoche, using part of one thing to refer to a thing in its entirety. Both are extremely common and familiar; e.g., in the phrase “boots on the ground”, “boots” is synecdoche for “soldiers”, or you might use the phrase “having wheels” to refer to owning a car, or a news report might say “the White House did such-and-such” to refer to the president or people working in the White House (metonymy), or the US government as a whole (synecdoche). It’s not limited to formal speech, and indeed various slang expressions, like the “wheels = car” example above, make extensive use of it. (Given that this is in the context of Moses speaking to the entire Israelite nation, it suggests he expects them to understand that it’s a figure of speech and is not literally asking permission for he, himself, to pass, so the implication is that people of all social strata used such figures of speech back then similarly to how they do now.)

      As to Ramesses and the Kadesh inscriptions possibly using synecdoche, I’m no expert, but from reading around a bit the general opinion seems to be that, though he managed to squeak out a victory (or at least a draw) from the jaws of defeat via personal battle prowess, reorganizing a scattered unit, and luck (and the Hittites stopping to loot his camp), the account is (at least slightly) self-aggrandizing propaganda. But I’m not familiar with the text itself so I can’t say for certain.

      1. Qadesh is primarily a piece of propaganda, Rameses fucked up entirely by himself and consequently doomed the entire campaign, he managed to extricate himself and part of his forces from the worst scenario of that fuckup, but it wasn’t any kind of draw, the Egyptians lost, very badly and would never again attain control over their former Syrian tributaries, who would remain in the hands of the Hittites at least until the Collapse.

        The Qadesh inscriptions were first and foremost an attempt by Rameses to disguise his defeat as a draw using precisely his ability to salvage the immediate tactical situation. It seems to be, as you can guess very effective.

        1. IMO the main reason Ramesses kept harping on Kadesh was because he was naively proud of his own prowess and refused to admit it was in fact a disaster.

  11. I feel like there’s room for a ‘football manager’ type of premodern military game. In football manager sims, the players control over the actual matches in flight is limited to substitutions etc, and in the main, you select the individuals for the various roles, give them a plan of action then watch the match unfold.

  12. You mention “bronze or iron” muscle cuirasses; it was my impression that ancient ironworking was not up to shaping large pieces of iron so that these were in fact exclusively bronze (or boiled leather). Was I mistaken about this?

  13. How did the augury get transmitted to the soldiers? Was it the night before, that before the general held his speech, his priests would perform the reading and tell the assembled men what the will of the gods was? Or did only the officers watch and then tell their men the results?
    I can imagine bad omens would be rather difficult to deal with. Do you just wait a day and hope the next day you get better omens? Or do you just say “well the sacred chickens werent too hungry, but don’t worry, they were quite thirsty, lets go”

    1. As Bret is fond of saying, most people in the ancient world believed their own religion – if you knew that you were fated to lose if you fought a battle today, why would you try to fight it?

      More pragmatically, I once read a theory that omens developed as a method of randomization – if you occasionally do what the gods say you should do instead of what you think is the best move, that makes you unpredictable, and that can sometimes give better results than a nonrandom strategy. Which, from the perspective of a believer, would look like the gods are actually giving you good advice.

      1. I read something similar about hunting on the steppe; you have to guess the location of animals over the horizon, and if you’re too predictable the animals will eventually figure it out.

      2. “why would you try to fight it?” You should ask Publius Claudius Pulcher that. The drowning of the sacred chickens was a reference to Battle of Drepana. Where, when the sacred chickens refused to eat, he cast them overboard stating “if they refuse to eat then they shall drink” (at least that is what the legend says). I can imagine some would cast the advice of the gods aside if they feel confident enough.

        The unpredictability story reminds me of how some people in the jungle would decide what area to slash and burn for farming. They would observe the flight of birds. Research indicates the birds do not provide any useful information. Their flight is random, leading effectively to a randomisation of where they decide to farm that year

        1. That sounds quite a lot like slander of a defeated commander to me. After all, the Romans lost the battle, unless Wikipedia is lying to me!

          1. If you’re an ancient Roman who believes in your religion…

            Well, your ancestors have lost a lot of battles. Not as many as they’ve won, but still a lot. Most of those defeats aren’t very memorable.

            But by the gods, literally, will you remember that one time that the commander sacrilegiously disrespected the sacred chickens, ignored the omens, charged into battle anyway, and then got his butt kicked!

            That is the kind of story that results in pious Romans for centuries afterwards never disrespecting the omens and portents again.

            Also, if you and everyone around you is a Roman who believes in their religion…

            …The odds are that if you’re reckless enough to disregard the omens and throw the sacred chickens into the water to drown, you’re reckless enough to make other ill-advised and risky choices too. Pulcher might have lost the Battle of Drepana for some other very material reason- due to recklessness.

    2. Greeks and Romans didn’t use augury, they used haruspicy. And yes, there are many instances of bad omens leading to delayed battles, sometimes over and over again.

      1. Cf. the Battle of Plataea, where, according to Herodotus, the two sides delayed for almost two weeks due to bad omens.

  14. It’s interesting that you bring up the Battle of the Wilderness here. Generals in the Civil War mostly ordered from behind the lines, but it was difficult, if not impossible to control troops difficult terrain. This is something that I think games like TW also get wrong. Terrain is mostly a non-issue. However, thick woods control access and force soldiers to clear a path or fight in a very tricky situation.

    That’s why Longstreet was leading from the front, as did Thomas Jackson a year earlier: they couldn’t communicate with any of the soldiers otherwise and had no idea what was going on, since they couldn’t see anything. On the Union side, General Grant mostly waited it out, but he engaged in a conspicuous display of confidence to tell his own subordinates not to panic. That’s sort of the half-inverted form compared to the warrior-general, “I’m not worried, so you shouldn’t be worried” vs. “I’m a badass so we’re gonna win”.

    1. At the battle of Chattanooga, the actual commands that won the battle were issued by officers with the troops, and under fire like them. Grant asked Thomas what they were doing, and Thomas did not know and could only say they could be hard to stop when they got going.

      1. Good example.

        I also just thought of Chickamauga, where an annoyed subordinate of Rosecrans technically obeyed orders, but in a deliberately improper way and created a gap in the lines. Then Longstreet, more-or-less by accident, happened to march his command right through it and caused a rout. The battle quite probably would have been inconclusive or even an outright Union victory if not for one staff officer behaving childishly! It’s certainly not what Rosecrans intended.

        1. OTOH, Shiloh turned to a Union victory when Grant returned, saw it was going on, and organized the rally to counterattack.

          1. And is another good example of the advantages and hazards of mid-19th C lead-from-the-front generalship: A. S. Johnson managed to push his troops pretty hard on the first day but also got shot and died.

            “Some of these critics claim that Shiloh was won when Johnston fell, and that if he had not fallen the army under me would have been annihilated or captured. […] Commanding generals are liable to be killed during engagements; and the fact that when he was shot Johnston was leading a brigade to induce it to make a charge which had been repeatedly ordered, is evidence that there was neither the universal demoralization on our side nor the unbounded confidence on theirs which has been claimed.” (Grant, Memoirs)

          2. One of the Confederate problems was that the men had been issued five days’ rations and been delayed for two. Some, finding food cooking on fires, stopped to eat it before fighting. This did not help.

          3. Colonel William T. Shaw of the 14th Iowa said of the Battle of Shiloh: “They outgeneraled us, but we outcolonelled them.”

          4. Grant’s response to a bad situation was to fight harder. No wonder he became Lincoln’s favorite general.

      2. Correction, Granger not Thomas made the comment about their being hard to stop.

        But, for added fun, apparently orders got garbled and some officers thought they had been ordered to take the ridge. At least, that’s what they said, after, and it could be true.

        OTOH, taking the rifle pits was a death trap, so some certainly took the initiative.

    2. To clarify my because I dashed off this comment without editing:

      “This is something that I think games like TW also get wrong. Terrain is mostly a non-issue. However, thick woods control access and force soldiers to clear a path or fight in a very tricky situation.”

      I mean that in games, terrain is often no more than bit of set dressing over a mostly-flat battlefield. Troops will basically move as quick over a road as a field, and it’s pretty much *all* open field. Even what should be thick forests don’t change much since it doesn’t affect your soldiers, except maybe the cavalry and even then not much.

      However, in reality terrain is a major factor, so much so that generals build battle plans around it (and still do!). Certainly in a pre-modern army, you most likely don’t want to attack in dense terrain like that, but you might be happy to defend it with skirmish troops, or even exploit it for cover while moving troops. Roman armies sometimes built causeways through almost-impassable marshes just to gain a strategic advantage. Bad terrain tends to cut communications and visibility, which can be a fatal weakness for an army before the telegraph.

      1. And famously during General Sherman’s Carolina Campaign, his army was able to advance through the Salk Swamps during the rainiest season of the year- something the Confederates had dismissed as impossible- by building its own roads and bridges as it went at the rate of ten miles a day.

        1. Military history has taught me that you should never, ever depend on bad terrain deterring an attack! And that combat engineers are an incredible advantage.

      2. Very true. A lot of World War II was fought for observation points, or limited by rivers. John Masters (Gurkha officer in the Indian Army) observed that skirmishes on the Afghan frontier gave you a life-or-death education in terrain (as the Pathans were famously good at using it), and several Japanese attacks that failed against his command in Burma would have carried through if they had been Afghans.

        1. During the Spanish Civil War, the Nationalist regulares similarly had a reputation for being able to make use of every little undulation in the ground to move around without being seen and launch sudden attacks on their enemies.

      3. Exactly how much terrain affects tends to vary over games (I do note that even in modern TW’s, commanding a fight in a forest can be a huge headache, though obviously much easier than it would be in reality) Troy, for instance, deliberate had much more impactful terrain mechanics and tied them to the weight class of troops (eg. light units were much more effective in rough terrain)

      4. One reason why the troops charged Missionary Ridge contrary to orders was terrain. The rifle pits they had been ordered to take were death traps ,but the terrain as they climbed provided much better shelter.

        OTOH, the units got so confused that some soldiers just obeyed whatever officer they could see.

  15. Wargames do this much better than video games. I recall how well The Gamers’ ACW Brigade series models this so very accurately. Orders you give must be written down, as precisely as possible, then sent by messenger and accepted by subordinate: each turn, you roll to see if an order arrives (most of the time there are significant delays) and once it reaches the brigade commander, then you roll again to see how he reacts (with his command stats affecting that roll), from going along to slowly implementing to loose cannon (ignoring it totally and doing a completely different thing). Other games since must have replicated or improved on this system.

  16. In the French comedy Kaamelott, a spoof on King Arthur, there are a number of battle scenes where Arthur is trying to order his small offscreen army around. Battle flags (the soldiers don’t look, and if they do, they don’t remember the meaning), horns rung (nobody pays attention or hears them), and when all else fail, shouting at them and the heavens in frustration. As for prebattle plans… let’s just say some of his subcommanders are really not up to the task and the scouting is about 50/50.

  17. Dr. D,

    You mention signal flags won’t work because the army is facing front, but couldn’t you just have a soldier (the commander maybe?) Whose job it is to look backwards on occasion so they can see what flag is raised?

    1. The difficulty with this is that once you get into the actual fight it is very hard to keep track of time and also relatively dangerous to turn your back to the enemy, even if you are not in the front.

  18. Are you planning to say more about just how subordinate your “subordinate” commanders might be? They are usually fellow aristocrats of similar status to yourself, and unless – or even if – you are actually their king, they have their own interests & agendas. So even if you do manage to solve the logistical problem of telling them what you want them to do, they may or may not have a strong interest in actually doing it.

    Presumably less of an issue for relatively strong centralised empires like the Romans or Ottomans, and more for little fractious west European mediaeval kingdoms?

    1. During the initial stages of the Imjin War, two Japanese commanders ignored all their orders and set off on a race to see who could be the first to capture Seoul. It worked out because the Japanese were hardened veterans and able to defeat their Korean opposites, but with stronger opponents it could very well have ended in disaster.

  19. Anybody in here played the old game Centurion?

    As far as I remember; in that game, which can be roughly described as an ancestor to Total War series, the key parameter of your commander was the strength of his voice. (Not 100% true to what our host describes, but bear with me.) In battle, you could only issue orders to these troops which fell within a given range from your commander. Outside of it, they could only advance until they either forced enemies off the field or themselves fled.

  20. RE imprecision of command and information. While these are absent from video games generally there are a lot of interesting systems for representing this in table top.

    In particular the Tactical Combat Series has a fun if elaborate system. Each player makes a plan by drawing on a printed copy of the game board, they must then adhere to this plan for several turns before changing it and any new plan takes several turns (lengthening with complexity) to prepare and communicate before it can be executed. In the simplest variant both players have full access to the board and chose how their units execute their plans. But ideal play has a game master or two subordinates who must execute the plans as provided without asking for clarification and only the game master/subordinates has access to the game board and must report unit positions and outcomes to the players. Hex coordinates can never be specified. The upshot is that while orders cannot be ignored or lost (fanciful orders will not be followed ofc) they can be misinterpreted, carried out imprecisely, or based on poor information. It’s a pain to get the group and the time to play but it can be a lot of fun.

  21. Random thought: Was this why Hannibal was so successful? I think we lack a detailed account of his fighting in Spain, but in Italia, at least, it seems that he carefully prepared before each battle and basically knew how it would go. Once the fighting started, he almost never needed to control things. The battle was effectively won once the Romans blundered into things, because he already had flanking forces ready.

    It also seems that until modern warfare, having a good cavalry commander who can keep his head and control the horsemen was an *extremely* powerful determiner of victory. Not always, but winning the cavalry engagement and being able to quickly re-organize and then support the ongoing battle, destroy the opposing camp, etc was a massive advantage.

    Also, a third question: The Mongolian and Manchu military seem to be something of an exception, as they are usually able to regroup very easily, break off and re-assemble, and coordinate over very long distances.

    1. I imagine the Mongolians and Manchus were mostly doing that while opposing armies with very much less mobility than themselves. That’s quite an advantage when you want to retreat and regroup.

      1. Steppe warfare was an extension of the great hunts, where units spread over great distances drove animals towards a prepared killing ground. Flags and drum calls coordinated the affair. Not only was everyone well-rehearsed, but severe criticism (at the least) attached to failure to perform one’s part. A nomad-on-nomad clash was many small units attacking then dropping back, while local commanders fed in others – a game of charge, retreat, feigned retreat to trap a small unit, exploit a break. Because it was fluid there was room to change on the fly – IF everyone knew what to do. Constant practice and iron discipline.

    2. The Mongolians at least had a roman style C&C which enabled them to take advantage of their mobility.

    3. My impression is that Hannibal’s army in Italy consisted largely of troops and officers who had been with Hannibal for a long time and felt they had good reason to trust his judgment. That personal relationship may have done a lot to ensure that his men stuck with the plan.

      Likewise, just because a general’s ability to control an army in battle is often limited to “make a plan and stick with it” doesn’t mean the general can’t win a lot of battles by usually having a better plan than the enemy.

  22. This (and the previous series) are reminding me of a great fun DOS game I must’ve played in like 1992, where the player gave turn-by-turn orders to a small squad of robot soldiers with different capabilities. The orders get issued–without knowing where the opponent might be–and can’t be changed during the combat resolution phase until the next round (call it 10 seconds of simulated time or something). I can’t for the life of me remember the name or find the game, but it definitely feels more like the conditions of an early commander than the typical perspective for even squad-combat games…

  23. Realistically constrained pre-modern battlefield command obviously isn’t going to be satisfying for fans of highly detailed real-time tactical gaming, but it seems like it could work well as a sort of mini-game within a game that’s *mostly* about grand strategy. Or maybe that’s just me—when I was a kid playing one of the very first *Total War* games, I was way more into the grand strategy aspects than the real time tactical simulations.

    1. Realistically pre-modern generalship doesn’t sound like any fun at all. Given the lack of reliable intelligence and the near impossibility of controlng a battle once joined. I’m not at all surprised that ancient generals leaned heavily on haruspices and omens to manage the uncertainty.

  24. Found my first-ever typo! Should be “Battle of the Wilderness” not “Battle of the Wildness”

  25. Of course this sort of pure coordination model is common in tactical video games which only infrequently put the player-as-general on the battlefield (or even if the ‘general’ of the army is represented on the battlefield, the survival of that figure is in no way connected to the player’s ability to coordinate the army).

    …except when the General unit dying is an instant Game Over, in which case most players will leave the General safe at one end of the battlefield, unless they’re engaging in some unorthodox strategy or the General has a lot of buffs.

    Trying to think of types of video games that could more accurately represent this side of generalship, and I’ve got three ideas.
    Just model all these difficulties, and make the game less about executing careful strategies and more about trying to wrestle with the limitations of command. Think of Children of a Death Earth, but limited by sociology and biology rather than physics. Or if you’re not familiar with an obscure game from several years ago, think of QWOP and other games built around obtuse controls.
    A visual-novel-style game that’s all about the leadership side of generalship, in particular about maintaining relationships with competent subordinates and the common soldier, with a battle system that tests how well you can use the people and trust you’ve cultivated (and avoid major blunders like sending your cavalry to charge a unit of spearmen in good order).
    A cross between Dynasty Warriors and Mount and Blade, with a side of DMC. You need to fight really good to convince your retinue and/or army to trust that you can lead them to victory. Silly, but it covers generalship themes not often covered, and would probably have a larger audience than the deeper but more niche ideas.

    1. I’m sorry that my list of ideas wound up as a big block of text. There was supposed to be some HTML list formatting that I guess WordPress doesn’t allow in comments.

    2. Surgeon Simulator: Total War. You spend ten minutes viscerally butchering a lamb, and then get to watch an noniteractive cutscene of the battle going on.

    3. Chris Crawford wrote a computer game “Excalibur” in the 1980s which did try to make the player as Arthur combine both the leadership aspect of managing relationships between the knights of the round table and the battlefield aspects of being a general. It didn’t do well, possibly because he was trying to run it on computers that measured memory and disk storage in kilobytes.

      But also possibly because the different aspects appeal to different types of player, so combining them into a single game for one person is almost guaranteed to annoy that person. Some people enjoy the diplomatic people management, some people prefer micromanaging battle units. It’s true that many people can enjoy both, but my experience is that not many game players will enjoy both *within the same game*.

      Where this kind of wide ranging game play does seem to work are the massively multiplayer online games. In EVE Online or World of Warcraft any individual player can probably find something that appeals to them, whether that is leading the clan raids or managing business deals with another megacorporation.

      1. Eve online, in fact, is an interesting case (at least the period i’m familiar with, 2008-2015).

        While communication is very easy (and made even easier by out-of-game voice coms), large scale warfare involves fleet commanders that must be on the battlefield to know what happens and give order, and “sniping” them is a legitimate enough strategy that fleets have a pre-determined chain of command to tell who is going to call the shots. It also involves a form of feodal “generaling” in that a fleet commander must, in order to have a fleet to command, convince other players of joining him rather than do anything else (including “play a better game”), through promises of fun, loot, storytelling or, on the contrary, getting kicked out of the space-guild if they don’t come often enough.

        And while it offers perfect information at all time about what is going on in the battlefield, there is significant clutter that can impede the parsing of said information, and any order can be mis-interpreted by any of the fleet member, especially when it’s more complex than “follow X, target Y, and press F1 to fire all the guns” (tho, tbh, it rarely gets more complex than that for the grunts, because even then they manage to fuck it up).

        And, of course, intelligence gathering via spies in the other fleet/space-guild, with it’s unreliability when counter-spying disseminate false informations.

        Man, I miss Eve. And would never play it again.

      2. I’d love to try an Excalibur-like game that wasn’t made with the technical constraints and design philosophies of 1983. Might try Excalibur anyways, see if I can find something that clicks in there.

        Anyways, the idea with the middle idea wasn’t to staple a visual novel and an RTS together, so much as it was to have a visual novel where battles are mechanically handled basically the same way as other conflicts—through a combination of the relationship values you’ve developed with characters and immediate “dialogue choices”. I can think of a few games in my Steam library that flirt with that idea, without focusing on the player character’s generalship. (As well as some visual novels which are stapled to an abstracted strategic sort of game, like Black Closet. Possibly Hanako Games’s best release to date.)

  26. I can’t help but think that a small number of decision points could be modeled both pretty accurately *and* in a fun way in a text-based game.

  27. Very interesting post. A good corollary would be a serious study of what mounted and foot formations in the pre-gunpowder era (and most of the post-gunpowder era too) were actually capable of. I’ve studied this topic with reference to the ancient military manuals – Arrian, Aelian, Asklepiodotus, Vegetius and others – and my own take is that formations were far more limited in their ability to manoeuvre than is commonly the case in popular wargames.

    Take wheeling: battlelines NEVER wheeled as lines. To change orientation, a line would need to form column, accomplished by square-shaped subunits (Roman century, Spartan pentecosty, Macedonian syntagma) wheeling simultaneously left or right to form an instant column which then moved off, the frontmost subunit wheeling in 90 degree increments (never less) followed by the subunits behind, until the column reached a new position and orientation where the subunits together wheeled 90 degrees to form a line again.

    Only trained infantry could form column from line and vice versa during a battle. Most infantry didn’t have this degree of training so once they formed a line at deployment all they could really do was advance.

  28. There’s quite a few computer wargames with interesting takes on command and control that I’m surprised more people haven’t mentioned, though I guess they mostly deal with modern warfare and are generally less mainstream.

    For pre-modern warfare there’s Pike & Shot/Sengoku Jidai/Field of Glory II, which are turn-based tabletop-like affairs that still let you order everybody around like in Total War but play very differently because units are generally unwieldy, difficult to turn, sensitive to morale shocks, and basically out of your control once they’re in contact or pursuit.

    I also have to mention Graviteam Tactics, Command Ops, and Flashpoint Campaigns, which, as far as I can tell, do pretty great jobs of modeling WWII and Cold War era command and control and its limitations, mainly via dynamic OODA loops and (usually) reliable AI control. In Graviteam Tactics, for instance, for your commands to actually reach units they need to go through runners, flares (visible to both sides!), radio transmissions, people shouting at each other, and/or telephone wires laid in real-time by actual dedicated units on the map. As such, you’re incentivized to micromanage only when necessary or when your communications network allows for it.

    So I think taking control away from the player can have its own appeal if it’s suitably evocative and makes for interesting decisions. And if the player in question was bad at micromanagement to begin with…

  29. A lot of this reminds me of the old (1993) game Fantasy Empires.

    Before a battle you’d choose where to put your units but you had to do it blind.

    Then during battle you could give very general orders to the whole army (charge, retreat, barrage, or skirmish) and then directly control the one dude you were personally controlling.

    To get your army to do stuff you could toggle through the units and move people into position one dude at a time while the AI controlled all the rest. You could also toggle an option to make nearby soldiers follow around the one dude you were controlling.

    You ability to micromanage your army was limited enough that often your best bet was to take control of your best unit, try to kill a lot of the enemy with that one unit and mostly leave the rest of the army up to the AI.

    The limited amount of micromanagement and how a lot of the battle was just you killing dudes with the one dude you were controlling made it a lot of fun. Sadly the game is hard to play now since you have to trottle the CPU to make the speed right and for some reason this slows down more units than others, specifically wizards csn lob fireballs with a ridiculous rate of fire even after you trottle the CPU while other units get slowed down properly.

  30. There’s a series of fantasy novels I occasionally read as fun junk-food reading, which are all about the exploits of a bunch of telepathic people on supernatural horses… Anyway, all these people are in service to the crown of a particular kingdom, they are loyal, brave, and reliable, and the military trusts them and respects their authority. Hence one of their jobs is to telepathically relay orders from the generals to military units in battle, and then to relay information from the units back to the generals. This being a fantasy universe, their enemies have magic communications, too, but never anything that can really be effectively used to the extent Our Heroes do.

    Yet somehow the kingdom is always in peril of being defeated by its enemies despite the fact that essentially have telepathic radio communication and their enemies don’t, which I had not thought about until reading this post. They should be rolling over their enemies and running the entire continent by now.

    1. If you’re referring to the Valdemar series, the Kingdom of Valdemar is secure most of the time because it has repeatedly thrashed attempted invasions. It helps that Valdemar is protected against most actual magic (as opposed to psionics which is a somewhat different thing). It’s ideologically opposed to conquest which is why it hasn’t pressed its advantage against it’s neighbors. And the times it is truly threatened are usually extraordinary- like the time evil sorcerers turned almost the entire population of a country into mind-controlled robots who would march and fight until they dropped dead.

      1. True, and the books of course only cover periods where the kingdom is in peril. But just in the lifetime of Selenay (who is only in, what, her 50s? at the latest point the books have reached) they have been at existential risk from Karse/the Tedrels once and Hardon 2-3 times (I’ve lost track) which seems like a lot for a large and prosperous country with such a massive communication advantage.

        (I will say I think the fact that Valdemar seems to confuse “we don’t fight wars of conquest” with “every time our neighbors attack us, we’ll fight back as little as necessary and just leave them to regroup and come at us again and again” is a rather silly aspect of the series.)

        1. Well of course Valdemar is impossibly ideal because of its founding premise: that angels in disguise reliably vet an elite of nearly incorruptible saints. Moreover only the homeland itself is protected against magic which makes venturing into foreign lands much riskier.

        2. I mean, what should Vakdemar do then? They have a pretty strict (and pretty modern) moral coed that largely holds throughout all levels of power. What can they do to prevent another attack that doesn’t violate that code? They can’t disarm the populace because the civilians need the blacksmiths as much if not more. They can’t conquer the place or instill a regime that’s more agreeable. They can’t just kill everyone. So what shoudl they do to secure themselves?

          1. I’m not a historian, so I could be wrong about this. If I had to guess, though, I’d guess that in real life we probably don’t see a lot of countries with a moral code that precludes doing anything but fighting strictly defensive battles, and I would guess that this is exactly why.

            So, yeah, I guess it’s maybe not surprising that a country that lets their enemies have unlimited tries to defeat them is constantly fighting despite a large military advantage. Still, I’d think they’d be doing better in the individual battles than they often seem to be.

        3. There’s an obscure Nintendo DS strategy RPG, Rondo of Swords, that is… not very good. But it does explore this line of thought. There’s one path of the game where you’ve finally repelled the enemy invasion and liberated your country. There is actually an option to just stop there and declare victory, which is encouraged by one of your trusted advisors. If you do, roll credits! You win! If you don’t and decide to counter-invade to conquer the conquerors (which is probably “canon”), the enemy gets increasingly desperate, turning to dark mages to resurrect the dead and the like, steps they wouldn’t have taken before, and there’s a bunch of carnage and the Sympathetic Trusted Advisor who told the main character not to do this ends up dying to make sure the player feels some pain too.

  31. There is an interesting commentary on this in some stories in the Fantasy genre.

    In about the only way the concept of instant generalship would work was if you actually had magic.

    An example of this being Belgarath the Sorcerer at the Battle of Vo Mimbre where the ability to shapeshift means he can fly and literally get an eagle’s eye view of the battlefield, then speak directly into the minds of others.
    (For the more speculatively inclined something similar is used in the story Saruman of Many Devices, to be found on Spacebattles.com)

  32. Since (the excellent) Ultimate General: Civil War, there’s been another game that’s come out, Grand Tactician: Civil War, that’s in the TW vein of having a macro strategy layer plus a UGCW-style battle mode with the brigade as the basic unit of maneuver. One very cool complication it has in the battle mode that addresses some of the things you’ve mentioned is order delay – when you give an order, unless the relevant unit is within bugle-call range of its commander, the order has to be written up and transmitted by a messenger on horseback. Even better, orders have to be pushed down the chain of command – if you give an order to an entire corps, messengers will go out from the corps commander to the individual division commanders, who will then send out their own messengers to their brigades. Indvidual officers can feud with other and even disobey orders as a result. It’s just a tick further in the grognard direction from UGCW, but still comprehensive and accessible.

  33. All due respect to the author, but I think he’s viewing the incident at the Wilderness through dung-colored glasses. “We’d really rather you not charge with us, it’s not your place” and “We really don’t want you to die” are not mutually exclusive opinions. (IIRC, that particular attack actually succeeded in carrying out its objective.) Sorry, but it’s not “Lost Cause” mythology to acknowledge that most of Lee’s soldiers actually liked him.

    It should also be noted that the Chancellorsville/Wilderness was a very atypical one for the ACW, and that Jackson, Longstreet, and Lee normally commanded from the rear. One could argue that they should have done as Grant did and accepted the fact that they weren’t going to be able to really know what was going on instead of riding forward to get shot, but the fact was that corps commanders on both sides were killed or wounded on a regular basis. At Gettysburg alone, three of the seven Union infantry corps commanders were killed or wounded, while the Antietam campaign saw two Union corps commanders killed, and James McPherson, commander of the Army of the Tennessee, was killed at the Battle of Atlanta. IOTW, this lead-from-the-front mentality is neither universal to nor unique to Confederate generals in the ACW, which means that you’re going to need another explanation for it besides “enslaver-planter culture” (whatever THAT is).

    1. “Enslaver-planter culture,” or its military aspect, would be more or less what a more Confederate-sympathizing source would call “Southron valor” or “chivalry.” The concept of command and authority popular among the plantation-owning aristocracy of the Southern states, in which the elite ‘planter’ class were among the bravest and finest men ever to live, so valorous and bold that they would fight regular duels to assert their personal honor, even in peacetime.

      This was basically the American-South version of Romantic culture. See also remarks on Southern aristocrats being huge fanboys of the works of Sir Walter Scott. See also the widespread conviction on the part of many in the Southern elite that they and their soldiers (most of whom, of course, were not of that social stratum) were simply much better men than the Union Army could raise against them.

      1. A combination of arrogance and sheer idiocy.
        As it turned out factory workers and new immigrants could and did kick southern gentlemen’s asses.

        1. Quite frequently, at that, when properly led. That last bit was a more of a problem in the East than in the West, though, for reasons I have never been able to determine. (Though some of it was probably also due to the relative ability of the Confederate leadership in each area. Lee’s merits compared to Grant’s and Sherman’s have been exaggerated. Compared to John Pemberton and Braxton Bragg…not so much.)

          1. I have more military merit than Braxton Bragg. So does everybody on this board. Why do we have a fort named for him?

          2. Bragg deserves a lot of the opprobium heaped upon him, but the fact is that he was a perfectly capable junior officer, and much like McClellan he was very good at preparing armies to fight.

            Unfortunately, he also had the soul of a bureaucrat. Just to give you an idea of what kind of man he was, at one point in his prewar military career, he was the quartermaster for the fort he was stationed at, and he commanded one of the units there.

            Now, a venal man might have used the opportunity to make sure his unit was supplied better than everyone else. A normal, honest man would have simply supplied his unit the same as everyone else and just skipped some of the paperwork.

            Bragg would write letters to himself asking for supplies, and would then write letters back explaining why he could or couldn’t provide them.

            He would have made an excellent quartermaster general or secretary of war, but he did not need to be commanding an army.

            As to why a fort is named after him? Because reasons.

          3. He maintained an official correspondence with himself? Okay, that’s just nuts!

          4. Why did the US Army end up with a fort named for Braxton Bragg? Well, the fort was established in North Carolina in 1918, when Woodrow Wilson was president. And that tells you a lot.

            My bet would be that the fort got named for General Bragg for the exact same reason that a great many other monuments lionizing Confederate military leaders were going up in the period from the 1880s up through the 1920s and on. Namely, as a move by very consciously white supremacist Southern politicians (including Wilson) to assert that while the South might have lost the Civil War, its cause had won the peace.

            We can find clear evidence of this in unveiling speeches for some of the Confederate Civil War monuments that went up elsewhere in the South, though I doubt there’s anything quite like it for Fort Bragg. This is also illustrated by how many monuments there are to various other Confederate generals, but how few such monuments there are to James Longstreet, one of the very few surviving Confederate officers who put real effort (including risking his life) to oppose white supremacist violence in the South after the war ended.

            https://www.thedailybeast.com/the-confederate-general-who-became-a-race-traitor

      2. So were non-Confederate senior officers who died or were wounded in action also acting under the influence of “enslaver-planter culture”?

        1. Maybe? Culture doesn’t generally tightly follow state boundaries. But more critically, enslaver-planter culture wasn’t the only one which might cause leading from the front even after the time for it has mostly passed.

          I admit I do wonder how much the ‘leading from the front’ has just moved down the chain due to the inability to even have any chance to see the general on a massive modern battlefield. I don’t think you hear much about Lieutenants commanding combat units staying that far back, and certainly more junior NCOs classically engage in such actions.

          1. As I understand it, the west expects officers to be somewhat back, and if the Lieutenant is personally engaging in combat instead of directing people something has gone wrong. They’re still expected to show coolness under fire and give the general impression they would definitely go forwards to personally grenade the machine gun nest if the tactical situation called for it, but if they have to actually prove they’d do that there’s been either a gross tactical error or a serious failure of cohesion.

          2. There’s still the general principle that the troops don’t know what’s going on outside of their very limited field of vision, so they’re going to be looking to the officers as the people who know what’s going on and will panic if the officers look like they’re panicking, but actual personal combat prowess is no longer a major expectation of commissioned officers.

          3. Maybe? Culture doesn’t generally tightly follow state boundaries. But more critically, enslaver-planter culture wasn’t the only one which might cause leading from the front even after the time for it has mostly passed.

            As other posters have indicated, Confederate officers’ behaviour in this regard was about in line with that of most mid-19th-century officers from pretty much any Western army, so unless Southern American culture was way more influential than hitherto suspected, I don’t think it has anything to do with “enslaver-planter culture”. As for it being anachronistic, I think that I’d generally trust actual contemporary officers’ assessments of what was warranted over that of a bunch of online commentators who’ve never even fought in a modern war, much less an 1860s-era one.

  34. Medieval Total War II’s unit descriptions claim that some of the knightly units may charge without orders. I’ve never actually seen it happen so I don’t know if it’s a genuine risk or not though.

  35. You wouldn’t want to watch a whole RTS battle play out when you can only give a few commands, but I’ve seen these double-blind choices a lot in board games like Kemet or Scythe. For example, both players in a battle know the size of each army, and they simultaneously choose a card from their hand to represent their battle plans.

    1. Interesting, the “auto battler” genre has been starting to come into vogue in the video game world recently, with games like Dota Underlords where you (so I gather, I haven’t played it) place units before each battle and then let them fight it out automatically until one side wins. Maybe it won’t be too long before we see the ability to give limited orders added and get some more historically-correct combat simulators.

  36. Lee was a wildly aggressive soldier, a fact missed by many at the beginning of the war because of his gentlemanly manner and interest in fortification. Usually he kept himself in hand but occasionally he had to be brought back to reality by those around him. By civil war times the private soldiers seem to have grasped that their general belonged at the rear managing the battle not playing hero in the front lines. On the other hand demonstrating a desire to get stuck in probably didn’t hurt a commander’s image, as long as he let himself be disuaded.

  37. “So one my well instead read it” – may.
    “Strategoi (generals) in Athens interesting seem” – interestingly

    As others have noted, “having to make a plan effectively blind and then mostly just hope it goes well” is the Dominions \ Conquest of Elysium model.

  38. Needless to say that means that giving any complex order to a unit already engaged or about to be engaged is going to mean starting by signalling retreat and then attempting to regroup the unit; regrouping an already retreating unit is one of the most difficult tasks on a battlefield and is rarely performed successfully in an unplanned fashion (even in an planned fashion it goes wrong as often as it goes right).

    I’m curious, how exactly would a unit retreat when engaged in melee combat? I’m guessing that turning their backs to the enemy they’re currently fighting would be a pretty bad idea, so wouod they walk backwards? If so, would they be able to maintain formation, or did they just accept that the formation would fall apart when retreating, and that they’d have to re-form afterwards? Also, was there anything stopping the enemy from following them? If not, how exactly would retreating improve things?

    1. Using melee weapons is really physically tiring. If the other line does not give in the first push, both sides will disengage, they will fall back a bit. If – big if – they are practiced and confident, the first rank can step back while the second or third keep the line. The trick is to maintain the line while moving away.

      1. So basically, insofar as there is a ‘trick’ since the maneuver cannot be performed reliably, the ‘trick’ to disengaging from an enemy in melee combat is to wait until mutual exhaustion forces the two sides’ combatants to fall back for a little breathing space, then back away slowly while brandishing a lot of sharp pointy metal at the enemy to discourage them from trying to chase you?

        1. Basically, yes. But note that some weapon-systems just cannot to this. A phalanx is eight rows of men in close order with 18′ poles, the first four or five lowered and pointing forwards. It goes one way – forwards (and everything gets out of the way or skewered). A line of archers can change direction of fire, but not move and fire effectively. Sword or spear guys can more easily fall back.

        2. But “back away slowly”, done right, sounds like not the whole unit stepping backward in lockstep, but a conveyor belt: front line moves back through the back lines with the second line holding ground, then the second line moves back, etc.

    2. Retreating in good order from a superior enemy has been called the most difficult maneuver. For good reason.

    3. Bear in mind that under most circumstances in shock combat you’re dealing with a unit that has a number of ranks- it’s only relatively late in gunpowder warfare that lines as thin as two or three ranks become used. And when you’re engaged in close shock combat, there’s a physical press as the front ranks interact with each other, most articulately with the “push of pike”. So you can pull back the disengaged ranks a bit, and then the front ranks are driven back a bit, and you can fight a retreat that way.

      But of course you’re thinning the actual line, and if the line breaks open, things all go to shit for the unit. Which is a key part of why retreat and regroup is such a difficult task- it puts everyone under stress to do things perfectly in a situation where they have very limited control.

      Another option (which is how the Roman system worked) is to have a second line and let the first line break away in a less controlled manner and fall back through holes in the second line, where they can then regroup behind the first, which is still deeply risky business. (And the in the triplex acies you have a third line of reserves in the event that this system fails.)

      But the main reason why retreating could improve things is that shock combat is physically and morally exhausting and it also takes time for an attacking unit to work itself back up to being able to get back into the press after it breaks away, even if it’s still in reasonably good order, which is by no means guaranteed. (Of course, attacking a crumbling unit is a different prospect entirely, which is another factor in why a retreat was intrinsically risky- the unit needs to look strong as it falls back, and also not break open during the retreat.) Going back to the push of pike, pike units would break apart from combat and take time to recover and cool down before getting back to fighting.

      While the tactics aren’t identical, the basics of fighting with lines and columns remains fairly similar throughout what Dr. Devereaux calls the “second military system”, so you can get an idea of how some of these maneuvers worked on a physical level by looking at a surviving infantry or cavalry drill manual from the 19th century, mainly the process of getting into formation and moving in formation.

      1. “But the main reason why retreating could improve things is that shock combat is physically and morally exhausting and it also takes time for an attacking unit to work itself back up to being able to get back into the press after it breaks away, even if it’s still in reasonably good order, which is by no means guaranteed.”

        The Romans had a system of Men fighting for a time before moving to the back with the row behind them moving forward into the fight as shown briefly here:

        https://www.youtube.com/watch?v=J7MYlRzLqD0

        Constant troop rotation allowed troops to rest for a time and allowed for recovery.

  39. At Agincourt, Henry V was supposed to have dressed a number of his knights in his own colors as decoys, with quite a few of them dying in battle. I wonder what the morale effect of this was, assuming it happened.

    1. If it’s generally known that there are several decoy kings on the field the effect could be very good. The army wouldn’t panic if they see their leader fall because odds are it’s not him.

      1. But on the other hand the inspirational aspect of watching your King fighting would be much reduced if odds are that the man battling bravely in front of you is just some minor noble–it seems like you may as well just stay behind the lines at that point.

  40. Regarding the Wilderness, I agree with 60guilders – it took place in scrub forest with no lines of sight worthy of the name, which may have given Lee the impetus to lead his own charge out of simple frustration in not being able to see what was going on. I’ll also point out that most of the general officers in the ACW on both sides went to the same school, West Point, and got their formative experiences in the same conflicts – particularly the Mexican-American Land Grab. Near-suicidal bravery in high command was commonplace on both sides, with obvious consequences. I’ll leave out the nearsighted Confederate general/bishop who rode up to the wrong formation and got blasted out of his saddle and just comment that even that guy’s own troops didn’t think he should have been in command, but he was amiable enough to avoid getting fragged.

  41. Complete ahistoricity aside, in Braveheart the Battle of Stirling actually gives a really good example of what a “leadership” general might actually do it a battle. Wallace has exactly two parts to his plan: allowing the cavalry to flank by appearing to flee and hiding his pikes until the charge is committed, and both are conveyed prior to to the battle. During the battle he executes the two orders, waving his arms for for the cavalry to “retreat” and yelling to raise pikes. In both cases the forces are primed to receive the orders, in that they were told when to expect them, the cavalry are watching him at the appropriate time for his signal and the infantry are likewise listening intently for his command at the moment they have been primed for. After the cavalry is broken he “gives” the order to charge by just charging himself. From that point on he is committed and gives no additional orders.

    What he does do at all points is be very conspicuous in his bravery. He shows up with his retinue, tall, hairy and blue; and gives a rousing speech. Of course, most of the men have no idea what he’s saying, but they can see it has impressed those who have. They know who he is at this point, or at least suspect, and his impressive reputation. In the narrative the majority of these men specifically showed up for battle due to the rumor that William Wallace might be there. He rides up and down the line leading a cheer, takes a short break to talk shit (which is also observable by his men! Again they cant hear his words, but by riding around and physically challenging the kings envoy he gives a very visible signal of his disrespect and willingness to fight!), then returns and dismounts, setting himself front and center of the infantry. His men, who are backed up on a hill can thus easily see that he is sharing their danger and commitment.

    So while the technical aspects of the fight and the historical aspects of everything else are pretty much entirely fantasy, I would argue in this scene they got generalship pretty much correct. I suspect this might be because a “Hollywood” idea of a brave leader squares up pretty well with what Leadership cultures expect from generals, both being rooted in conspicuous courage a martial prowess.

  42. You mention the “Take Command!” series for having an order delay. There is another videogame series that makes some attempt at a more accurate presentation of command and control – the game Spartan and later game Legion’s Arena.

    Both games have tactical battles that look similar to Total War, but you must line your armies up at the battle start and give them orders. There are the default “charge forward” orders, but your units generally won’t flank the enemy with those orders. For flanks you need to command them to do a flanking run, but once again you can only give them a rough idea of how far back to go before turning to perform the flank, so it is a rough tool.

    In Spartan that is all you can do, but Legion’s Arena adds a “General” unit (a guy on a horse). During the battle you can give new orders to units within a set distance of your “General” unit, but there is also a cooldown timer so you can only give so many new orders at a time. This goes some way to making your suggested “reserves” system worth doing.

    You still have the overhead drone view of the battle, and the “General” command radius is rather large, but still an interesting point of comparison.

  43. A modern warfare example of how orders can go wrong is the Charge of the Light Brigade. Army Commander Lord Raglan wanted a Russian unit driven away from some guns they had captured. A staff officer, Captain Nolan, transmitted the message orally, indicated the wrong Russian unit, and had their commander send the Light Brigade against a dug-in Russian artillery unit.

  44. I bet a game involving the “warrior-hero general” archetype could be fun, although totally different from the sort of top-down RTS game one usually thinks of.

    Imagine: first you position the units of your army on the battlefield, but without knowing where the enemy is; this is akin to the battle planning the night before. Maybe they can be given broad commands that the AI tries to act out autonomously, but you don’t get to control them in real time once the battle starts.

    Then you start the battle and the perspective shifts to first-person (or over-the-shoulder third-person), and you dive into the melee as the highly-visible general. Defeat enemy units or capture points of interest, and your nearby troops will rally around you and fight more fiercely; get hit or knocked down too often and they will lose morale and run. You affect your army’s morale, but not as a passive area bonus; it’s linked to how well you fight. Maybe you could even do risky-but-flashy maneuvers to display bravery even more conspicuously but at the risk of taking more damage if you do get hit.

    A game like this would probably be tough to balance and make it a satisfying play experience, but it seems possible.

    1. As a technical matter it probably wouldn’t be too hard to transform Fire Emblem Warriors or its ilk into something like this. Make the random generic guys you beat up for points capable of fighting each other and have a “morale” charge meter to go with the other combat charge meters affected by such-and-so that gives your generic guys bonuses. Then have generic guys form on you and the other characters on your team, who you already have some limited capacity to direct, and have a small slate of command stats to differentiate them.

      The main problem from a fun perspective would probably be that you could lose the battle because of events on the opposite side of the battlefield beyond your ability to meaningfully influence, which is already a bit of a problem with those games and would be amplified if having four hundred generic spearmen in your path actually meant something.

  45. Was it expected that the “warrior-hero” type general be an elite fighter who could kick any normal soldier’s ass in a straight fight? That’s how they’re described in the Iliad, and I’m guessing Alexander was like that, but I don’t get that impression from, say, Gaius Julius. Especially as they get older, it seems like it would become obvious that they’re not quite as elite as they used to be, or even on par with the regular soldiers. Then again, maybe that didn’t matter, if they were just waving their sword around at skirmishing range while hoping to not get an arrow in the knee. Was there a “retirement plan” for generals who got old, or were they just expected to fight until they die like Alexander?

    1. Roman warfare was not of the heroic type. Romans counted on discipline and teamwork not individual heroics, isn’t that right, Bret? From Republican times Roman generals were consuls or former consuls meaning they were in their forties at least and often older.

      1. The Romans imposed more severe penalties for failure to retreat promptly on being ordered to than for cowardice, because discipline was that important.

        1. Didn’t they decimate units for cowardice?

          Roman armies were more about disciplined teamwork than individual heroics, but there was some degree of expectation that generals would display conspicuous gallantry, and Augustus reputedly once personally led an escalade. And there was the spolia opima, the highest honor for a general claimed from defeating the enemy commander in single combat. So there was at least some expectation of personal valor even by army commanders.

          1. Not that it was necessarily common. There’s, like, two well-sourced examples of the spolia opima counting the one Augustus rules-lawyered Crassus out of.

      2. Roman warfare was not of the heroic type. Romans counted on discipline and teamwork not individual heroics, isn’t that right, Bret? From Republican times Roman generals were consuls or former consuls meaning they were in their forties at least and often older.

        Yes and no. Roman generals acted in a less heroic style than, say, Alexander the Great, but the Roman system of sword-armed heavy infantry left more room for common soldiers to perform individual acts of heroism than the Macedonian phalanx.

    2. Personally I suspect that, especially as people got older, a lot of the fighting was done by their elite personal guard. Alexander was presumably a decent fighter given how many battles he made it through, but also he was leading the heavy cavalry that formed the main striking force of his army, and had the very best of those at his side.

      Also, depending on the era, your generals might be warrior-aristocrats who spend a lot of their off-time training, and the bulk of your regulars do not do that. So taking on a forty-something general might be like playing football against Tom Brady. Once they got really old, they’re still aristocrats and there’s plenty of non-military positions of power and influence.

    3. You can stay very competitive with spear, sword and shield into your 50s. A lot is just reading the moves – and the more experience you have, the better you get. Alexander’s Silver Shield veterans were kicking arse twenty years on, veteran legions would see off the young bloods in style, a lot of Norse saga champions are formidable into old age. My son’s 60-something fencing teacher would give five 20 minute lessons to fit teenagers in a row, and leave them each drenched and panting – he told me the secret was he knew at all times what was going to happen next.

  46. Keegan tried to differentiate the styles of command much as described in this article, and presented Grant as a post-heroic general, canny enough to stay off the firing line while being close enough to personally view the fighting and totally eschewing finery . Alexander was the Heroic, pre-gunpowder Ur-example.

    I played the old map-and-cardboard counter game Terrible Swift Sword (Gettysburg) back in the day, and during one game my Confederate opponent put A P Hill right up front during the first day’s battles, where he was swiftly shot. The result, according to the rules, was 15 minute’s immobility for the entire I Corps, after which Pender took over and all was as before. It does seem as if this was both too swift a result and too little lasting consequence

  47. You write, “Because units are not in melee contact, engagements are less decisive (units advance, receive fire, break, fall back and then often reform to advance again; by contrast a formation defeated in a shock engagement tends not to reform because it is chased by the troops that defeated it), giving more space for units to maneuver in substantially longer battles. Moreover, units under fire can maneuver, whereas units in shock generally cannot, which is to say that a formation receiving musket or artillery fire can still be controlled and moved about the field, but a unit receiving sword strikes is largely beyond effective command except for ‘retreat!’”

    I’m curious how that might interact with a pulse model of fighting. Specifically, if the pulse model is accurate, one might expect some more room for maneuver, controlled withdrawals, and command even after a unit is committed. Is the pulse model wrong, or is there something else going on that makes engagement sticky?

    1. Part of it is the critical need to maintain the line. A unit cannot falls back if it opens the way for neighbouring units to be flanked, but coordinating that in the press is really hard. Late Republic/early Imperial Romans with sword and shield, triple lines and distributed command (and extensive training) could manage it. Not many others.

Leave a Reply to YrroCancel reply